2012年6月英语四级模拟题带答案(一至五)

2012年6月英语四级模拟题带答案(一至五)

ID:82792417

大小:380.00 KB

页数:64页

时间:2023-09-24

上传者:180****8072
2012年6月英语四级模拟题带答案(一至五)_第1页
2012年6月英语四级模拟题带答案(一至五)_第2页
2012年6月英语四级模拟题带答案(一至五)_第3页
2012年6月英语四级模拟题带答案(一至五)_第4页
2012年6月英语四级模拟题带答案(一至五)_第5页
2012年6月英语四级模拟题带答案(一至五)_第6页
2012年6月英语四级模拟题带答案(一至五)_第7页
2012年6月英语四级模拟题带答案(一至五)_第8页
2012年6月英语四级模拟题带答案(一至五)_第9页
2012年6月英语四级模拟题带答案(一至五)_第10页
资源描述:

《2012年6月英语四级模拟题带答案(一至五)》由会员上传分享,免费在线阅读,更多相关内容在教育资源-天天文库

大学英语四级考试模拟题一PartⅠWriting(30minutes)Directions:Forthispart,youareallowed30minutestowriteashortessayonthetopic:ONE–OFFCHOPSTICKS.Youshouldwriteatleast120wordsfollowingtheoutlinegivenbelowinChinese:1.你和你的朋友在餐厅就餐,你的朋友对餐厅提供的非一次性筷子不满。2.餐厅的负责人解释说这是为了节约资源,并保证这些餐具已经严格消毒;3.你对此事的看法和采取的做法。  One-offChopsticks注意:此部分试题在答题卡1上。PartⅡReadingComprehension(SkimmingandScanning)(15minutes)Directions:Inthispart,youwillhave15minutestogooverthepassagequicklyandanswerthequestionsonAnswerSheet1.Forquestions1-7,choosethebestanswerfromthefourchoicesmarkedA,B,CandD.Forquestions8-10,completethesentenceswiththeinformationgiveninthepassage.考试就注意:此部分试题请在答题卡1上作答;8-10题在答题卡1上。  PlasticBagPollutionTheuseofplasticbagshasincreasedatanalarmingratesincetheybecamepopularinthe1980s.Bigblackbinliners,plasticcarrierbagscarryingadvertisinglogos,clearsandwichbagsandavarietyofotherforms.They’relightweight,handyandeasilydiscarded.Tooeasilydiscarded.Justtakealookaroundyou.Plasticbagscanbeseenhangingfromthebranches,flyingintheair,settledamongstbushes,andfloatingonrivers.Theyblockupdrainscausingwaterandsewagetooverflowandbecomethebreedinggroundsofbacteriathatcausediseases.Plasticbagsaredifficultandcostlytorecycleandmostenduponlandfillsiteswheretheytakearound300yearstodecay.Theybreakdownintotinytoxicparticlesthatpollutethesoilandwaterwaysandenterthefoodchainwhenanimalsaccidentallyconsumethem.Buttheproblemssurroundingwasteplasticbagsstartlongbeforetheydecay.Ourplanetisbecomingincreasinglyendangeredbyourover-useofplasticbags.ProductionofPlasticBagsPlasticbagsaremadefromethylene(乙烯),agasthatisproducedasaby-productofoil,gasandcoalproduction.Ethyleneismadeintopolymers(聚合体)—chainsofethylenemolecules—calledpolyethylene.Thissubstance,alsoknownaspolythene,isusedtoproducearangeofitems,includingplasticbags.Youhaveprobablynoticedtwotypesofplasticshoppingbags—thelighter,filmybagsyougetfromsupermarketsandfoodoutlets,andtheheavierbagsfromotherretailoutlets,likeclothingstores.Thesupermarketbagsaremadefromhighdensitypolyethylene(HDPE),whilethethickerbagsaremadefromlowdensitypolyethylene(LDPE).UnlikeHDPE,LDPEcannotberecycled.Whileplasticbagsmaynotbethemosthightechapplicationofplasticstechnology,itiscertainlyoneofthemostprevalent.AccordingtoCleanUpAustralia,Australiansuseinexcess

1of6billionplasticbagsperyear.Iftiedtogetherthesebagswouldformachainthatislongenoughtogoaroundtheworld37times.Morethanhalfofthesebags(3.6billion)aremadefromHDPE.DangerstoSeaLifePlasticbagsarenowamongstthetop12itemsofwastemostoftenfoundalongthecoastlines.Animalsandseacreaturesarehurtandkilledeverydaybydiscardedplasticbags—adeadturtlewithaplasticbaghangingfromitsmouthisn’tapleasantsightbutmistakingplasticbagsforfoodiscommonplaceamongstmarineanimals.Plasticblockstheirintestines(肠道)andleadstoslowstarvation.Othersbecomeentangledinplasticbagsanddrown.Becauseplasticbagstakehundredsofyearstobreakdown,everyyearourseasbecome“home”tomoreandmorebagsthatfindtheirwaytherethroughoursewersandwaterways.Everybagthat’swasheddownadrainduringrainfallendsupinthesea;everybagthat’sflusheddownatoiletendsupinthesea;everybagthat’sblownintoariverwillmostlikelyendupinthesea.Onedisquietingfactstemmingfromthisisthatplasticbagscanbecomeserialkillers.Onceananimaldietofplasticbags,itdecaysatamuchfasterratethanthebag.Oncetheanimalhasdecomposed,thebagisreleasedbackintotheenvironmentmoreorlessintact,readytobeeatenbyanothermisguidedorganism.Theincrediblyslowrateofdecayofplasticbagsalsomeansthateachbagweusecompoundstheproblem,becausethebagssimplyaccumulate.PollutionTaxes&BansDifferentcountrieshaveadoptedarangeofapproachestodiscouragetheuseofplasticbagsinanattempttocutdownonthenumberofbagsfindingtheirwayintotheenvironment.InSouthAfricaforinstance,whereanestimatedeightbillionplasticbagsareusedannually,thegovernmenthasimplementednewregulationsthatwillseeonlythicker,moredurableplasticbagsproduced.Aswellasmakingthemmoresuitableforreuse,itishopedthattheextracostassociatedwiththeirproductionandsupplywillpreventretailersgivingthehigherqualitybagsaway,makingtheiruseamoreexpensiveoptionforconsumers.TheuseofplasticbagsisbeingdiscouragedinotherplacessuchasSingaporeandTainwan,whilethetaximposedontheuseofplasticshoppingbagsinIrelandhasresultedintheuseofplasticshoppingbagsbeingreducedby90%injustsixmonths.Priortothe15eurocentperbagtax,itwasestimatedthat1.2millionplasticshoppingbagswerebeinghandedoutinIrelandperyear.Themoneyraisedfromthetaxwillbeusedtofundenvironmentalinitiatives.Whileanythingthatlowersourconsumptionisgood,whywaituntilwe’rehitfinanciallybeforewechangeourhabitswhenchangesaren’tdifficulttomake?Howabouttakingpreviouslyusedbagswithyounexttimeyougototheshops?Orevenbetter—turnbacktimeanddoasgrandmadidandtakeabamboobasketwithyoueverytimeyougoshopping.Shop-ownerswouldratheryouusetheirbagsasthey’reaconvenientandcheapformofadvertising,butwhat’smoreimportant,shareholderprofitsortheenvironment?Plasticbagscanbere-cycledalthoughonlyaboutoneinevery200everfindsitswaytoarecyclingunit.Findoutifthere’sare-cyclingbinnearyouand,ifnot,askyourlocalauthorityforone.GreenhouseGasesSomecountrieshaveintroducedsocalled“environmentallyfriendlyplasticbags”thatarebiodegradable(可生物降解).Thesebagstakeaboutthreeyearstobreakdownintopracticallynothingandwhilethatsoundslikeanattractivesolution,thetruthisthattheprocessofbreaking

2downthesepetrol-basedbagscausescarbontobecomemethane(甲烷)whichisagreenhousegas.It’salsopossibletoget“plastic”bagsmanufacturedfromcorn.Thesebreakdownveryquicklyandgiveoffnomoremethanethananothercornproductonlandfillsites.Unfortunately,they’remoreexpensivetoproduceandfewshopsusethem.GarbageBinsOnequestionthat’softenraisedinconnectionwiththeplasticbagdilemmaiswhatshouldbeusedtolinebinsifnotplasticbags?Toanswerthat,let’sgobackintimetowhenplasticbagswereyettobecomecommonplace.Whatdidwedo?Forone,wehadfarlessgarbage.Goodshadmuchlesspackagingthanisthecasetodaysowedidn’tfillourbinsasquickly.Peelings,eggshells,teabagsandcoffeegroundswereallusedascompost(肥料)forcrops,aswasanypaperthatwasn’tneededforlightingfires.Whatwasleftwentintoanunlinedbinwithanythingdirtyorwetbeingwrappedinnewspaperfirst.Ifwechoosecarefullywhenshoppingsoasnottobringhomemorepackagingthannecessary—there’sreallynopointinbringithomejusttothrowitstraightinthebinwhenweunpack,theamountofwastethatgoesintothekitchenbinwillbehalved,atleast.IndividualEffortByrefusingtouseplasticbags,youcanmakeahugedifferencetothepollutionproblem.Rememberthateachpersonusesabout83bagsayear.Iftherearefourpeopleinyourfamily,that’s332plasticbagslesseveryyear.That’s332bagslessthatwill:—releasetoxinsintothegroundwaterfromlandfillsites—stayintheenvironmentforhundredsofyears—getintothefoodchainthroughanimals—wasteenergyduringthemanufacturingprocess—killanyoftheestimated100,000marineanimalsthatdieeachyearofplasticpollutionTheseareall-importantfactorsthathaveaprofoundaffectonourenvironmentandthecreaturesweshareourplanetwith.Shouldwereallyputourownselfishneedsbeforetheneedsofeverythingaroundusnowandthelivesoffuturegenerations?That’suptoyoutodecide.1.Manythick,clothing-carryingplasticbags,madefrom________,canhardlyberecycled.A.highdensitypolyethyleneB.lowdensitypolyethyleneC.oilandcoalD.gasandcola2.Theannualuseofplasticbagsin________exceeds6billion.A.SingaporeB.AustraliaC.IrelandD.Taiwan3.Plasticbagscanbecomeahorribleserialkillerbecauseof________.A.theirtoxicparticlesB.theirlightweightC.theirincrediblyslowrateofdecayD.theiroverusebypeople4.InSouthAfrica,inlinewithanewgovernmentregulation,________willbebannedfromproduction.A.thicker,moredurableplasticbagsB.plasticbagssuitableforreuseC.thinner,short-livingplasticbagsD.thicker,higher-qualityplasticbags5.________,thoughfriendlytotheenvironment,costtoomuchinmanufacturetowidelyused.A.ThepetrolbasedbagsB.Themethane-releasingbagsC.Thecarbon-releasingbagsD.hecorn-basedbags6.Whatisbotheringpeopleinconnectionwithplasticbagdilemma?

3A.Thequestionofwhatshouldbeusedtoreplaceplasticbagsasgarbagebinliners.B.Thefactthatonlyoneinevery200plasticbagsendupbeingrecycled.C.Theincreasingamountofgarbage.D.Thehighcostinvolvedinproducingenvironmentallyfriendlybags.7.It’sestimatedthateveryyearabout________arekilledbydiscardedplasticbags.A.10,000animalsB.100,000seaanimalsC.100,000landanimalsD.10,000wildlife8.Thefinaldestinationofmanycarelesslydiscardedplasticbagsis____________________.9.InIreland,followingthecollectionofa15eurocentperbagtax,plasticbagusagehasdroppedby________.10.Theso-calledenvironmentallyfriendlessbagsaremuchquickertobreakdown,butwillreleaseagreatdealof______________________________.PartⅢListeningComprehension(35minutes)SectionADirections:Inthissection,youwillhear8shortconversationsand2longconversations.Attheendofeachconversation,oneormorequestionswillbeaskedaboutwhatwassaid.Boththeconversationandthequestionswillbespokenonlyonce.Aftereachquestiontherewillbeapause.Duringthepause,youmustreadthefourchoicesmarkedA,B,CandD,anddecidewhichisthebestanswer.ThenmarkthecorrespondingletteronAnswerSheet2withasinglelinethroughthecentre.注意:此部分试题请在答题卡2上作答。11.A.ThemanmightbeabletoplayintheWorldCup.B.Theman’sfootballcareerseemstobeatanend.C.Themanwasoperatedonafewweeksago.D.themanisafanofworld-famousfootballplayers.12.A.Workoutaplantotightenhisbudget.B.Findouttheopeninghoursofthecafeteria.C.ApplyforaseniorpositionintherestaurantD.Solvehisproblembydoingapart-timejob.13.A.Afinancialburden.B.Agoodcompanion.C.Arealnuisance.D.Awell-trainedpet.14.A.Theerrorswillbecorrectedsoon.B.Thewomanwasmistakenherself.C.Thecomputingsystemistoocomplex.D.Hehascalledthewomanseveraltimes.15.A.Heneedshelptoretrievehisfiles.B.Hehastotypehispaperoncemore.C.Heneedssometimetopolishhispaper.D.Hewillbeawayforatwo-weekconference.16.A.Theymighthavetochangetheirplan.B.Hehasgoteverythingsetfortheirtrip.C.Hehasaheavierworkloadthanthewoman.D.TheycouldstayinthemountainsuntilJune8.

411.A.Theyhavetowaitamonthtoapplyforastudentloan.B.Theycanfindtheapplicationformsinthebrochure.C.Theyarenoteligibleforastudentloan.D.Theyarenotlateforaloanapplication.12.A.Newlawsareyettobemadetoreducepollutantrelease.B.Pollutionhasattractedlittleattentionfromthepublic.C.Thequalityofairwillsurelychangeforthebetter.D.It’lltakeyearstobeingairpollutionundercontrol.Questions19to22arebasedontheconversationyouhavejustheard.13.A.Enormoussizeofitsstores.B.Numerousvarietiesoffood.C.Itsappealingsurroundings.D.Itsrichandcolorfulhistory.14.A.Anancientbuilding.B.Aworldofantiques.C.AnEgyptianmuseum.D.AnEgyptianMemorial.15.A.Itspowerbillreaches$9millionayear.B.Itsellsthousandsoflightbulbsaday.C.Itsuppliespowertoanearbytown.D.Itgenerates70%oftheelectricityituses.16.A.11,500.B.30,000.C.250,000.D.300,000.Questions23to25arebasedontheconversationyouhavejustheard.17.A.Transferringtoanotherdepartment.B.Studyingaccountingatauniversity.C.Thinkingaboutdoingadifferentjob.D.Makingpreparationforherwedding.18.A.Shehasfinallygotapromotionandapayraise.B.Shehasgotasatisfactoryjobinanothercompany.C.Shecouldatlastleavetheaccountingdepartment.D.Shemanagedtokeepherpositioninthecompany.19.A.HeandAndreahaveprovedtobeaperfectmatch.B.Hechangedhismindaboutmarriageunexpectedly.C.Hedeclaredthathewouldremainsingleallhislife.D.HewouldmarryAndreaevenwithoutmeetingher.SectionBDirections:Inthissection,youwillhear3shortpassages.Attheendofeachpassage,youwillhearsomequestions.Boththepassageandthequestionswillbespokenonlyonce.Afteryouhearaquestion,youmustchoosethebestanswerfromthefourchoicesmarkedA,B,CandD.ThenmarkthecorrespondingletteronAnswerSheet2withasinglelinethroughthecentre.注意:此部分试题请在答题卡2上作答。PassageOneQuestions26to29arebasedonthepassageyouhavejustheard.20.A.Theyaremotorcyclesdesignatedforwatersports.B.theyarespeedyboatsrestrictedinnarrowwaterways.C.Theyarebecominganefficientformofwatertransportation.D.Theyaregettingmorepopularasameansorwaterrecreation.21.A.Waterscooteroperators’lackofexperience.BVacationers’disregardofwatersafetyrules.

5C.Overloadingofsmallboatsandothercraft.D.Carelessnessofpeopleboatingalongtheshore.11.A.Theyscarewhalestodeath.B.Theyproducetoomuchnoise.C.Theydischargetoxicemissions.D.Theyendangerlotsofwaterlife.12.A.Expandoperatingareas.B.Restrictoperatinghours.C.Limittheuseofwaterscooters.D.Enforcenecessaryregulations.PassageTwoQuestions30to32arebasedonthepassageyouhavejustheard.13.A.theyarestable.B.Theyareclose.C.Theyarestrained.D.Theyarechanging.14.A.Theyarefullyoccupiedwiththeirownbusiness.B.Notmanyofthemstayinthesameplaceforlong.C.Notmanyofthemcanwintrustfromtheirneighbors.D.Theyattachlessimportancetointerpersonalrelations.15.A.Countoneachotherforhelp.B.Giveeachotheracoldshoulder.C.Keepafriendlydistance.D.Buildafencebetweenthem.PassageThreeQuestions33to35arebasedonthepassageyouhavejustheard.16.A.Itmayproduceanincreasingnumberofidleyoungsters.B.ItmayaffectthequalityofhighereducationinAmerica.C.Itmaycausemanyschoolstogooutofoperation.D.Itmayleadtoalackofproperlyeducatedworkers.17.A.Itislessseriousincitiesthaninruralareas.B.Itaffectsbothjuniorandseniorhighschools.C.Itresultsfromaworseningeconomicclimate.D.ItisanewchallengefacingAmericaneducators.18.A.Allowingthemtochoosetheirfavoriteteachers.B.Creatingamorerelaxedlearningenvironment.C.Rewardingexcellentacademicperformance.D.Helpingthemtodevelopbetterstudyhabits.SectionCDirections:Inthissection,youwillhearapassagethreetimes.Whenthepassageisreadforthefirsttime,youshouldlistencarefullyforitsgeneralidea.Whenthepassageisreadforthesecondtime,youarerequiredtofillintheblanksnumberedfrom36to43withtheexactwordsyouhavejustheard.Forblanksnumberedfrom44to46youarerequiredtofillinthemissinginformation.Fortheseblanks,youcaneitherusetheexactwordsyouhavejustheardorwritedownthemainpointsinyourownwords.Finally,whenthepassageisreadforthethirdtime,youshouldcheckwhatyouhavewritten.注意:此部分试题在答题卡2上;请在答题卡2上作答。I’minterestedinthecriminaljusticesystemofourcountry.Itseemstomethatsomethinghastobedoneifweareto(36)________asacountry.Icertainlydon’tknowwhattheanswerstoourproblemsare.Thingscertainlyget(37)_________inahurrywhenyougetintothem.ButIwonderifsomethingcouldn’tbedonetodealwithsomeoftheseproblems.OnethingI’mconcernedaboutisourpracticeofputting(38)_________injailwhohaven’tharmedanyone.Whynotworkoutsomesystem(39)_________theycanpaybackthedebtstheyowesocietyinsteadof(40)________anotherdebtbygoingtoprisonandofcoursecomingoutthe(41)

6________ofhardenedcriminals.I’malsoconcernedabouttheshortprisonsentencespeopleare(42)________forseriouscrimes.Ofcourseonealternativetothisisto(43)________capitalpunishment.ButI’mnotsureIwouldbeforthat.I’mnotsureit’srighttotakeaneyeforaneye.(44)___________________________________________________________________________.Ialsothinkwemustdosomethingabouttheinsanityplea.Inmyopinion,anyonewhotakesanotherperson’slifeintentionallyisinsane.However,(45)_______________________________________________________________________.Itsaidofcoursethatapersonmayhavetospendtherestofhislifeor(46)____________________________________________________________________________.PartIVReadingComprehension(ReadinginDepth)(25minutes)SectionADirections:Inthissection,thereisapassagewithtenblanks.Youarerequiredtoselectonewordforeachblankfromalistofch0icesgiveninawordbankfollowingthepassage.Readthepassagethroughcarefullybeforemakingyourchoices.Eachchoiceinbankisidentifiedbyaletter.PleasemarkthecorrespondingletterforeachitemonAnswerSheet2withasinglelinethroughthecenter.Youmaynotuseanyofthewordsinthebankmorethanonce.Questions47to56arebasedonthefollowingpassage.  WhenRobertoFelizcametotheUSAfromtheDominicanRepublic,heknewonlyafewwordsofEnglish.Educationsoonbecamea47.“Icouldn’tunderstandanything,”hesaid.He48fromhisteachers,camehomeintears,andthoughtaboutdroppingout.  ThenMrs.Malave,abilingualeducator,begantoworkwithhimwhileteachinghimmathandscienceinhis49Spanish.“ShehelpedmestaysmartwhileteachingmeEnglish,”hesaid.Giventhechancetodemonstratehisability,he50confidenceandbegantosucceedinschool.  Today,heisa51doctor,runshisownclinic,andworkswithseveralhospitals.Everyday,heusesthelanguageandacademicskillshe52throughbilingualeducationtotreathispatients.  Roberto‘sstoryisjustoneof53successstories.Researchhasshownthatbilingualeducationisthemost54waybothtoteachchildrenEnglishandensurethattheysucceedacademically.InArizonaandTexas,bilingualstudents55outperformtheirpeersinmonolingualprograms.Calexico,Calif.,implementedbilingualeducation,andnowhasdropoutratesthatarelessthanhalfthestateaverageandcollege56ratesofmorethan90%.InE1Paso,bilingualeducationprogramshavehelpedraisestudentscoresfromthelowestinTexastoamongthehighestinthenation.  注意:此部分试题请在答题卡2上作答。[A]wonder[B]acquired[C]consistently[D]regained[E]nightmare[F]native[G]acceptance[H]effective[I]hid[J]prominent[K]decent[L]countless[M]recalled[N]breakthrough[O]automatically      SectionB来源:Directions:Thereare2passagesinthissection.Eachpassageisfollowedbysomequestionsorunfinishedstatement.ForeachofthemtherearefourchoicesmarkedA),B),C),D).Youshould

7decideonthebestchoiceandmarkthecorrespondingletteronAnswerSheet2withasinglelinethroughthecenter.PassageOneQuestion57to61arebasedonthefollowingpassage.  “Tear’emapart!”“Killthefool!”“Murderthereferee(裁判)!”  Thesearecommonremarksonemayhearatvarioussportingevents.Atthetimetheyaremade,theymayseeminnocentenough.Butlet’snotkidourselves.Theyhavebeenknowntoinfluencebehaviorinsuchawayastoleadtorealbloodshed.Volumeshavebeenwrittenaboutthewaywordaffectus.Ithasbeenshownthatwordshavingcertainconnotations(含义)maycauseustoreactinwaysquiteforeigntowhatweconsidertobeourusualhumanisticbehavior.Iseetheterm“opponent”asoneofthosewords.Perhapsthetimehascometodeleteitfromsportsterms.  Thedictionarymeaningoftheterm“opponent”is“adversary”;“enemy”“onewhoopposesyourinterests.”Thus,whenaplayermeetsanopponent,heorshemaytendtoeveryactionnomatterhowgross,maybeconsideredjustifiable.Irecallanincidentinahandballgamewhenarefereerefusedaplayer’srequestforatimeoutforaglovechangebecausehedidnotconsiderthemwetenough.TheplayerproceededtorubhisglovesacrosshiswetT-shirtandthenexclaimed,“Aretheywetenoughnow?”  Intheheatofbattle,playershavebeenobservedtothrowthemselvesacrossthecourtwithoutconsideringtheconsequencesthesuchamovemighthaveonanyoneintheirway.Ihavealsowitnessedaplayerreactingtohisopponent’sintentionalandillegalblockingbydeliberatelyhittinghimwiththeballashardashecouldduringthecourseofplay.Offthecourt,theyaregoodfriends.Doesthatmakeanysense?Itcertainlygivesproofofacourtattitudewhichdepartsfromnormalbehavior.  Therefore,Ibelieveitistimeweelevated(提升)thegametothelevelwhereitbelongs,therebysettinganexampletotherestofthesportingworld.Replacingtheterm“opponent”with“associate”couldbeanidealwaytostart.  Thedictionarymeaningoftheterm“associate”is“colleague”;“friend”;“companion.”Reflectamoment!Youmaysoonseeandpossiblyfeelthedifferenceinyourreactiontotheterm“associate”ratherthan“opponent”.注意:此部分试题请在答题卡2上作答。57.Whichofthefollowingstatementsbestexpressestheauthor’sview?A)Thewordspeopleusecaninfluencetheirbehavior.  B)Unpleasantwordsinsportsareoftenusedbyforeignathletes.  C)Aggressivebehaviorinsportscanhaveseriousconsequences.  D)Unfairjudgmentsbyrefereeswillleadtoviolenceonthesportsfield.58.Harshwordsarespokenduringgamesbecausetheplayers_______.  A)aretooeagertowin  B)treattheirrivalsasenemies  C)areusuallyshort-temperedandeasilyoffended  D)cannotaffordtobepoliteinfiercecompetitions59.Whatdidthehandballplayerdowhenhewasnotallowedatimeouttochangehisgloves?  A)Heangrilyhittherefereewithaball.  B)Herefusedtocontinuethegame.  C)Heclaimedthatrefereewasunfair.

8  D)HewethisglovesbyrubbingthemacrosshisT-shirt.60.Accordingtothepassage,playersinagamemay______.  A)kicktheballacrossthecourtwithforce  B)liedownonthegroundasanactofprotest  C)deliberatelythrowtheballatanyoneillegallyblockingtheirway  D)keeponscreamingandshoutingthroughoutthegame61.Theauthorhopestohavethecurrentsituationunsportsimprovedby________.  A)regulatingtherelationshipbetweenplayersandreferees  B)callingonplayerstousecleanlanguageinthecourt  C)raisingthereferee‘ssenseofresponsibility  D)changingtheattitudeofplayersonthesportsfieldPassageTwoQuestions62to66arebasedinthefollowingpassage.  IsthereenoughoilbeneaththeArcticNationalWildlifeRefuge(保护区)(ANWR)tohelpsecureAmerica’senergyfuture?PresidentBushcertainlythinksso.HehasarguedthattappingANWR’soilwouldhelpeaseCalifornia’selectricitycrisisandprovideamajorboosttothecountry’senergyindependence.Butnooneknowsforsurehowmuchcrudeoilliesburiedbeneaththefrozenearth,withthelastgovernmentsurvey,conductedin1998,projectingoutputanywherefrom3billionto16billionbarrels.  Theoilindustrygoeswiththehighendoftherange,whichcouldequalasmuchas10%ofU.S.consumptionforaslongassixyears.Bypumpingmorethan1millionbarrelsadayfromthereserveforthenexttwotothreedecades,lobbyistsclaim,thenationcouldcutbackonimportsequivalenttoallshipmentstotheU.S.fromSaudiArabia.Soundsgood.Anoilboomwouldalsomeanamultibillion-dollarwindfall(意外之财)intaxrevenues,royalties(开采权使用费)andleasingfeesforAlaskaandtheFederalGovernment.Bestofall,advocatesofdrillingsay,damagetotheenvironmentwouldbeinsignificant.“We’veneverhadadocumentedcaseofanoilrigchasingdeeroutontothepackice,”sayAlaskaStateRepresentativeScottOgan.  Notsofast,sayenvironmentalists.StickingtothelowendofgovernmentestimatestheNationalResourcesDefendsCouncilsaystheremaybenomorethan3.2billionbarrelsofeconomicallyrecoverableoilinthecoastalplainofANWR,adropinthebucketthatwoulddovirtuallynothingtoeaseAmerica’senergyproblems.Andconsumerswouldwaituptoadecadetogainanybenefits,becausedrillingcouldbeginonlyaftermushbargainingoverleases,environmentalpermitsandregulatoryreview.AsforANWR’simpactontheCaliforniapowercrisis,environmentalistspointoutthatoilisresponsibleforonly1%oftheGoldenState’selectricityoutput---andjust3%ofthenation’s.注意:此部分试题请在答题卡2上作答。62.WhatdoesPresidentBushthinkoftappingoilinANWR?  A)ItwillincreaseAmerica’senergyconsumption.  B)Itwillexhaustthenation’soilreserves.  C)Itwillhelpreducethenation’soilimports.  D)ItwillhelpsecurethefutureofANWR.63.WelearnfromthesecondparagraphthattheAmericanoilindustry_________.  A)showslittleinteresttappingoilinANWR  B)expecttostopoilimportsfromSaudiArabia  C)tendtoexaggerateAmerica’srelianceonforeignoil

9  D)believesthatdrillingforANWRwillproducehighyields64.ThoseagainstoildrillingANWRarguethat________.  A)itwilldraintheoilreservesintheAlaskanregion  B)itcandolittletosolveU.S.energyproblem  C)itcancauseseriousdamagetotheenvironment  D)itwillnothavemuchcommercialvalue65.Whatdotheenvironmentalistsmeanbysaying“Notsofast”(Line1,Psra.3)?  A)Don‘tbetoooptimistic.B)Don’texpectfastreturns.  C)Theoildrillingshouldbedelayed.  D)Oilexploitationtakesalongtime.66.ItcanbelearnedfromthepassagethatoilexploitationbeneathANWR’sfrozenearth________.  A)involvesalotoftechnologicalproblems B)remainsacontroversialissue  C)isexpectedtogetunderwaysoonD)willenabletheU.S.tobeoilindependentPartVCloze(15minutes)Directions:Thereare20blanksinthefollowingpassage.ForeachblanktherearefourchoicesmarkedA),B),C)andD)ontherightsideofthepaper.YoushouldchoosetheONEthatbestfitsintothepassage.ThenmarkthecorrespondingletteronAnswerSheet2withasinglelinethroughthecenter.注意:此部分试题请在答题卡2上作答。  Wisebuyingisapositivewayinwhichyoucanmakeyourmoneygofurther.The67yougoaboutpurchasinganarticleoraservicecanactually68youmoneyorcanadd69thecost.Takethe70exampleofahairdryer.Ifyouarebuyingahairdryer,youmight71thatyouaremakingthe72buyifyouchooseone73lookyoulikeandwhichisalsothecheapest74price.Butwhenyougetithomeyoumayfindthatit75twiceaslongasamoreexpensive76todryyourhair.Thecostoftheelectricityplusthecostofyourtimecouldwell77yourhairdryerthemostexpensiveoneofall.Sowhatprinciplesshouldyou78whenyougooutshopping?Ifyou79yourhome,yourcaroranyvaluable80inexcellentcondition,you’llbesavingmoneyinthelong81.Beforeyoubuyanew82,talktosomeonewhoownsone.Ifyoucan,useitorborrowittocheckitsuitsyourparticular83.Beforeyoubuyanexpensive84,oraservice,docheckthepriceand85isonoffer.Ifpossible,choose86threeitemsorthreeestimates.67.[A]form[B]fashion[C]way[D]method68.[A]save[B]preserve[C]raise[D]retain69.[A]up[B]to[C]in[D]on70.[A]easy[B]single[C]simple[D]similar71.[A]convince[B]accept[C]examine[D]think72.[A]proper[B]best[C]reasonable[D]most73.[A]its[B]which[C]whose[D]what74.[A]for[B]with[C]in[D]on75.[A]spends[B]takes[C]lasts[D]consumes76.[A]mode[B]copy[C]sample[D]mode77.[A]cause[B]make[C]leave[D]bring

1078.[A]adopt[B]lay[C]stick[D]adapt79.[A]reserve[B]decorate[C]store[D]keep80.[A]products[B]possession[C]material[D]ownership81.[A]run[B]interval[C]period[D]time82.[A]appliance[B]machinery[C]utility[D]facility83.[A]function[B]purpose[C]target[D]task84.[A]component[B]element[C]item[D]particle85.[A]what[B]which[C]that[D]this86.[A]of[B]in[C]by[D]fromPartVITranslation(5minutes)Directions:CompletethesentencesonAnswerSheet2bytranslatingintoEnglishtheChinesegiveninbrackets.87._____________________________(两天了都没有走出沙漠,又没有水喝),histhirstwasunquenchable.88.Bycontrast,Americanmothersweremorelikely________________(把孩子的成功归因于)naturaltalent.89.Ifshehadreturnedanhourearlier,Mary___________________________(就不会被大雨淋了).90.We’dbetterstruggleforthefuture_____________________________(而不是为过去而懊悔).91.Notuntilpeoplecancompletelytrustyou______________________________(你才能对他们产生积极的影响).

11大学英语四级考试模拟题二PartIWriting(30minutes)  Directions:Forthispart,youareallowed30minutestowriteacompositiononthetopicCompetition.Youshouldwriteatleast120wordsfollowingtheoutlinegivenbelowinChinese:  1.竞争使人们充满创造力,使人们更有效率。  2.竞争促使生产出更好的产品和提供更优质的服务。  3.竞争促进了社会的进一步发展。  Competition  PartIIReadingComprehension(SkimmingandScanning)(15minutes)  Directions:Inthispart,youwillhave15minutestogooverthepassagequicklyandanswerthequestionsonAnswerSheet1.  Forquestions1-7,mark  Y(forYES)ifthestatementagreeswiththeinformationgiveninthepassage;  N(forNO)ifthestatementcontradictstheinformationgiveninthepassage;  NG(forNOTGIVEN)iftheinformationisnotgiveninthepassage.  Forquestions8-10,completethesentenceswiththeinformationgiveninthepassage.  Earthquake  CanScientistsPredictKillerEarthquakes?  ThedatewasNovember23,1980.PeoplenearNaples,insouthernItaly,felttheEarthrollandshake.Earthquake!Suddenlybuildingscametumblingdown.Cracksappearedintheearth.Withinminutes,entiretownsweredestroyed.Thousandsofpeopleweredead.Thousandsmorewereinjured.  Asrescuerssearchedthroughtherubble,manypeoplemusthavewondered,ofonlythevictimshadknownaheadoftime,manylivescouldhavebeensaved.  Actually,anItalianscientistdidpredictthatsuchaquakewouldhappen.In1977Dr.M.CaputooftheUniversiteDegilesStudiinRomewarnedthatalargequakewouldsoonstriketheeastofNaples.Unfortunately,hecouldn’tpredicttheexacttimeanddateofthequake.  Dr.Caputomadehisgeneralpredictionaftertalkingwithscientistsat54earthquakemonitoringstationsthroughoutItaly.HelearnedthatmanyearthquakeshadrecentlyrockeddifferentareasaroundNaples.ButnonehadoccurredinoneparticularspoteastofNaplesformanyyears.Dr.Caputofeltthattheareawaslongoverdueforalargequake.Anditwas.  EarthquakeStrikesinGap  ThequakeoccurredinaregionthatDr.Caputocalledaseismicgap.Aseismicgapisanareainanactiveearthquakeregionwherenoearthquakeorseismicactivityhasbeenrecordedinalongtime.SeismicgapsarelocatedwheretwolargeplatesintheEarthhavebecomestuck.  Whentheplatesslidepasteachother,theysometimesbecamelockedinplace.Asimilarthinghappenswhenyoumakearunningleaponasidewalkwhilewearingsneakers.Whenyoulandonbothfeet,thesneakersgrabontoroughsurface.Frictiontendstoholdyourfeetbackwhiletherestofyourbodygoesforward.Youmayendupfallingflatonyourface.  Inthecaseofplates,however,theunevensurfacesbetweentheplatescausetheplatestoremainlockedinplaceforyears.Hugepressurebuildsupbehindeachplate.Periodically,a

12shudder,ortremor,isrecordedassomeofthisenergyisreleased.  Finally,afterabout50years,rockintheseismicgapeithersuddenlybreaksormovesunderthegreatstress.Thissuddenreleaseofenergysendsshockwavesthroughtherocklayersabove.Thegroundshakes,sidewalkscrack,andbuildingstumble.Amightyquakehasstruck.  GapsUsedToPredictQuakes  Manygeologistshaveusedwhatiscalledtheseismicgaptechniquetoaccuratelypredictearthquakes.ThetechniquewasfirstdevelopedbySovietearthquakeexpertDr.V.FodotovduringhisstudiesofancientandrecentJapaneseearthquakes.Dr.Fodotovwasmarkingthelocation,size,anddateofallknownquakesinJapanwhenhenoticedastrikingpattern.  AllmajorearthquakeswerefoundtooccurinonlyafewisolatedspotsinJapan.Eachofthesespots,henoted,experiencedamajorquakeonlyonceevery50to60years.Dr.Fodotovconcludedthatspotsthathadnthadaquakeinmorethan50yearswere“ripe”foraquake.TheRussianscientistnamedtheselocationsseismicgaps.  Inthepastseveralyears,geologistsfromothercountrieshavefoundseismicgapsinotherpartsoftheworld.Aftermakingdetailedstudiesofpastquakesintheseregions,thegeologistswereabletomakeanaccuratepredictionofwhenaquakewouldoccur.  HowDoAnimalsKnowWhenanEarthquakeIsComing?  Scientistswhotrytopredictearthquakeshavegottensomenewhelpersrecently—animals.  That’sright,animals.Scientistshavebeguntocatchontowhatfarmershaveknownforthousandsofyears.Animalsoftenseemtoknowinadvancethatanearthquakeiscoming,andtheyshowtheirfearbyactinginstrangeways.BeforeaChinesequakein1975,snakesawokefromtheirwintersleepearlyonlytofreezetodeathinthecoldair.Cowsbroketheirhaltersandtriedtoescape.Chickensrefusedtoentertheircoop.Allofthisunusualbehavior,aswellasphysicalchangesintheearth,alertedChinesescientiststothecomingquake.Theymovedpeopleawayfromthedangerzoneandsavedthousandsoflives.  Onetaskforscientiststodayistolearnexactlywhichtypesofanimalbehaviorpredictquakes.It’snotaneasyjob.Firstofallnoteveryanimalreactstothedangerofanearthquake.JustbeforeaCaliforniaquakein1977,forexample,anArabianstallionbecameverynervousandtriedtobreakoutofhisstall.Thehorsenexttohim,however,remainedperfectlycalm.It’salsodifficultattimestotellthedifferencebetweennormalanimalrestlessnessand“earthquakenerves”.Azookeeperoncecalledearthquakeresearcherstosaythathiscougarhadbeenactingstrangely.Itturnedoutthatthecathadanupsetstomach.  Asecondtaskforscientistsistofindoutexactlywhatkindofwarningstheanimalsreceive.Theyknowthatanimalssensefarmoreoftheworldthanhumansdo.Manyanimalscansee,hear,andsmellthingsthatpeopledonotevennotice.Somecandetecttinychangesinairpressure,gravity,orthemagnetismofEarth.Thisextrasenseprobablyhelpsanimalspredictquakes.  Agoodexampleofthisoccurredwithagroupofdogs.Theywerepennedupinanareathatwasbeingshakenbyaseriesoftinyearthquakes.(Severalsmallquakesoftencomebeforeorafteralargeone.)Beforeeachquakealowboomingsoundwasheard.Eachboomcausedthedogstobarkwildly.Thenthedogsbegantobarkduringasilentperiod.Ascientistwhowasrecordingtilequakeslookedathismachine.Itwasactingasthoughtherewerealoudnoisetoo.Thescientistrealizedthatthedogshadreactedtoaboomingnoise.Theyalsosensedthetinyquakethatfollowedit.Themachinerecordedboth,thoughhumansfeltandheardnothing.  Inthiscasetherewasamachinetomonitorwhatthedogsweresensing.Manytimes,

13however,ourmachinesrecordnothingoutoftheordinary,eventhoughanimalsknowaquakeiscoming.Theanimalsmightbesensingsomethingwesomeasurebutdonotrecognizeasawarning.Discoveringwhatanimalssense,andlearninghowtheyknowitisadangersignal,isajobforfuturescientists.  1.SincenoonehadpredictedtheprecisedateoftheearthquakestrikingeastofNaples,peopletheresufferedheavylossinthedestruction.  2.AseismicgapislocatedatthejunctionoftwointerlockingplatesintheEarth,andwherenoseismicactivityhasbeenrecordedforalongtime.  3.Fromthepassagewelearnthataregularstrikingpatterncanbefoundinanactiveearthquakeregion.  4.DuringanearthquakeinChina1975,cowsbroketheirhaltersandranawayfromtheirsheds.  5.AsitisusedinParagraph13,theword“cat”referstoatypicaldomesticcat.  6.Allanimalsbutmencannoticetinychangedinairpressure,gravity,orthemagnetismofEarth.  7.ThedogsmentionedinParagraph15hadsensedboththelowboomsandtheminorquakesfollowingthem.  1.[Y][N][NG]2.[Y][N][NG]3.[Y][N][NG]  4.[Y][N][NG]5.[Y][N][NG]6.[Y][N][NG]  7.[Y][N][NG]  8.Dr.CaputobasedhispredictionuponthefactthatlotsofearthquakeshadrecentlyoccurredinallareasaroundNaplesbutits___________.  9.Accordingtotheauthor’sinformation,every50yearsorso,amightyearthquakewillberecordedat___________________.  10.Chinesescientistsevacuatedpeoplefrom_____________aftertheyhadnoticedthestrangebehaviorofsomeanimalsaswellasphysicalchangesintheearth.PartⅣReadingComprehension(ReadinginDepth)(25minutes)  SectionA  Directions:Inthissection,thereisapassagewithtenblanks.Youarerequiredtoselectonewordforeachblankfromalistofchoicesgiveninawordbankfollowingthepassage.Readthepassagethroughcarefullybeforemakingyourchoices.Eachchoiceinthebankisidentifiedbyaletter.PleasemarkthecorrespondingletterforeachitemonAnswerSheet2withasinglelinethroughthecentre.Youmaynotuseanyofthewordsinthebankmorethanonce.  Questions47to56arebasedonthefollowingpassage.  WhatisitaboutAmericansandfood?Welovetoeat,butwefeel47aboutitafterward.Wesaywewantonlythebest,butwestrangelyenjoyjunkfood.We’re48withhealthandweightlossbutfaceanunprecedentedepidemicofobesity.Perhapsthe49tothisambivalenceliesinourhistory.ThefirstEuropeanscametothiscontinentsearchingfornewspicesbutwentinvain.Thefirstcashcropwasn’teatenbutsmoked.ThentherewasProhibition,intendedtoprohibitdrinkingbutactuallyencouragingmore50waysofdoingit.  Theimmigrantexperience,too,hasbeenoneofinharmony.DoasRomansdomeanseatingwhat“realAmericans”eat,butournation’sfoodhascometobe51byimports-pizza,say,orhotdogs.Andsomeofthecountry’smosttreasuredcookingcomesfrompeoplewhoarrivedhere

14inshackles.  Perhapsitshouldcomeasnosurprisethenthatfoodhasbeenamediumforthenation’sdefiningstruggles,whetherattheBostonTeaPartyorthesit-insatsouthernlunchcounters.Itisintegraltoourconceptsofhealthandevenmoralitywhetheronerefrainsfromalcoholforreligiousreasonsorevadesmeatforpolitical52.  Butstrongopinionshavenotbrought53.Americansareambivalentaboutwhattheyputintheirmouths.Wehavebecome54ofourfoods,especiallyaswelearnmoreaboutwhattheycontain.  The55infoodisstillprosperousintheAmericanconsciousness.It’snocoincidence,then,thatthefirstThanksgivingholdstheAmericanimaginationinsuchbondage(束缚).It’swhatweeat—andhowwe56itwithfriends.  [A]answer[B]result[C]share[D]guilty  [E]constant[F]defined[G]vanish[H]adapted  [I]creative[J]belief[K]suspicious[L]certainty[M]obsessed[N]identify[O]ideals  SectionB  Directions:Thereare2passagesinthissection.Eachpassageisfollowedbysomequestionsorunfinishedstatements.Foreachofthemtherearefourchoicesmarked[A],[B],[C]and[D].YoushoulddecideonthebestchoiceandmarkthecorrespondingletteronAnswerSheet2withasinglelinethroughthecentre.  PassageOne  Questions57to61arebasedonthefollowingpassage.  ItisnotoftenrealizedthatwomenheldahighplaceinsouthernEuropeansocietiesinthe10thand11thcenturies.Asawife,thewomanwasprotectedbythesettingupofadowry(嫁妆).Admittedly,thepurposeofthiswastoprotectheragainsttheriskofdesertion,butinrealityitsfunctioninthesocialandfamilylifeofthetimewasmuchmoreimportant.Thedowrywasthewife’srighttoreceiveatenthofallherhusband’sproperty.Thewifehadtherighttowithholdconsent,inalltransactionsthehusbandwouldmake,andmorethanjustaright;thedocumentsshowthatsheenjoyedarealpowerofdecision,equaltothatofherhusband.Innocasedothedocumentsindicateanydegreeofdifferenceinthelegalstatusofhusbandandwife.  Thewifesharedinthemanagementofherhusband’spersonalproperty,buttheoppositewasnotalwaystrue.Womenseemedperfectlypreparedtodefendtheirowninheritanceagainsthusbandswhotriedtoexceedtheirrights,andonoccasiontheyshowedafinefightingspirit.AcaseinpointisthatofMariaVivas.HavingagreedwithherhusbandMirotosellafieldshehadinherited,fortheneedsofthehousehold,sheinsistedoncompensation.Nonebeingoffered,shesucceededindraggingherhusbandtothescribetohaveacontractdulydrawnupassigningherapieceoflandfromMiro’spersonalinheritance.Theunfortunatehusbandwasobligedtoagree,asthecontractsays,“forthesakeofpeace.”Eitherthroughthedowryorthroughbeinghot-tempered,thewifeknewhowtowinherself,withthecontextofthefamily,apowerfuleconomicposition.  57.Originally,thepurposeofadowryisto_________.  [A]giveawomantherighttoreceiveallherhusband’sproperty  [B]helpawomantoenjoyahigherpositioninthefamily  [C]protectawomanagainsttheriskofdesertion

15  [D]bothAandC  58.Accordingtothepassage,thelegalstatusofthewifeinmarriagewas__________.  [A]higherthanthatofasinglewoman  [B]higherthanthatofherhusband  [C]lowerthanthatofherhusband  [D]thesameasthatofherhusband  59.WhydoestheauthorgiveustheexampleofMariaVivas?  [A]Toshowthatthewifesharedinthemanagementofherhusbandspersonalproperty.  [B]Toshowthatthewifecandefendherowninheritance.  [C]Toprovethatwomenhavepowerfulposition.  [D]Toillustratehowwomenwinherproperty.  60.ThecompensationMariaVivasgotforthefieldis____________.  [A]someofthelandMirohadinherited  [B]atenthofMiro’sland  [C]moneyforhouseholdexpenses  [D]moneyformMiro’sinheritance  61.Theauthor’sattitudetowardsMariaVivasis_____________.  [A]sympathetic[B]disapproval[C]indifferent[D]objective  PassageTwo  Questions62to66arebasedonthefollowingpassage.  Accordingtosociologists,thereareseveraldifferentwaysinwhichapersonmaybecomerecognizedastheleaderofasocialgroup.Inthefamily,traditionalculturalpatternsconferleadershipononeorbothoftheparents.Inothercases,suchasfriendshipgroups,oneormorepersonsmaygraduallyemergeasleaders,althoughthereisnoformalprocessofselection.Inlargergroups,leadersareusuallychosenformallythroughelectionorrecruitment.  Althoughleadersareoftenthoughttobepeoplewithunusualpersonalability,decadesofresearchhavefailedtoproduceconsistentevidencethatthereisanycategoryof“naturalleaders”.Itseemsthatthereisnosetofpersonalqualitiesthatallleadershaveincommon;rather,virtuallyanypersonmayberecognizedasaleaderifthepersonhasqualitiesthatmeettheneedsofthatparticulargroup.  Researchsuggeststhattherearetypicallytwodifferentleadershiprolesthatareheldbydifferentindividuals.Instrumentalleadershipisleadershipthatemphasizesthecompletionoftasksbyasocialgroup.Groupmemberslooktoinstrumentalleadersto“getthingsdone”.Expressiveleadership,ontheotherhand,isleadershipthatemphasizesthecollectivewell-beingsofasocialgroup’smembers.Expressiveleadersarelessconcernedwiththeoverallgoalsofthegroupthanwithprovidingemotionalsupporttogroupmembersandattemptingtominimizetensionandconflictamongthem.  Instrumentalleadersarelikelytohavearathersecondaryrelationshiptoothergroupmembers.Theygiveothersandmaydisciplinegroupmemberswhoinhibitattainmentofthegroupsgoals.Expressiveleaderscultivateamorepersonalorprimaryrelationshiptoothersinthegroup.Theyoffersympathywhensomeoneexperiencesdifficultiesandtrytoresolveissuesthatthreatentodividethegroup.Asthedifferenceinthesetworolessuggest,expressiveleadersgenerallyreceivemorepersonalaffectionfromgroupmembers;instrumentalleaders,iftheyaresuccessfulinpromotinggroupgoals,mayenjoyamoredistantrespect.

16  62.Whatdoesthepassagemainlydiscuss?  [A]Theproblemsfacedbyleaders.  [B]Howleadershipdiffersinsmallandlargegroups.  [C]Howsocialgroupsdeterminewhowillleadthem.  [D]Theroleofleadersinsocialgroups.  63.ThepassagementionsallofthefollowingwaysbywhichpeoplecanbecomeleadersEXCEPT_____________.  [A]recruitment  [B]formalelectionprocess  [C]specificleadershiptraining  [D]traditionalculturalpatterns  64.WhichofthefollowingstatementsaboutleadershipcanbeinferredfromParagraph2?  [A]Apersonwhoisaneffectiveleaderofaparticulargroupmaynotbeaneffectiveleaderinanothergroup.  [B]Fewpeoplesucceedinsharingaleadershiprolewithanotherperson.  [C]Apersoncanbestlearnhowtobeaneffectiveleaderbystudyingresearchonleadership.  [D]Mostpeopledesiretobeleadersbutcanproducelittleevidenceoftheirqualifications.  65.Inmentioning“naturalleaders”inLine7,theauthorismakingthepointthat____________.  [A]fewpeoplequalifyas“naturalleaders”  [B]thereisnoproofthat“naturalleaders”exist  [C]“naturalleaders”areeasilyacceptedbythemembersofagroup  [D]“naturalleaders”shareasimilarsetofcharacteristics  66.Thepassageindicatesthatinstrumentalleadersgenerallyfocuson___________.  [A]ensuringharmoniousrelationships  [B]sharingresponsibilitywithgroupmembers  [C]identifyingnewleaders  [D]achievingagoal  PartⅤCloze(15minutes)  Directions:Thereare20blanksinthefollowingpassage.Foreachblanktherearefourchoicesmarked[A],[B],[C]and[D]ontherightsideofthepaper.YoushouldchoosetheONEthatbestfitsintothepassage.ThenmarkthecorrespondingletteronAnswerSheet2withasinglelinethroughthecentre.  Ifyouweretobeginanewjobtomorrow,youwouldbringwithyousomebasicstrengthsandweaknesses.Successor67inyourworkwoulddepend,to68greatextent,69yourabilitytouseyourstrengthsandweaknessestothebestadvantage.70theutmostimportanceisyourattitude.Aperson71beginsajobconvincedthatheisn’tgoingtolikeitoris72thatheisgoingtoailisexhibitingaweaknesswhichcanonlyhinderhissuccess.Ontheotherhand,apersonwhoissecure73hisbeliefthatheisprobablyascapable74doingtheworkasanyoneelseandwhoiswillingtomakeacheerfulattempt75itpossessesacertainstrengthofpurpose.Thechancesarethathewilldowell.76theprerequisiteskillsforaparticularjobisstrength.Lackingthoseskillsisobviouslyaweakness.Abookkeeperwhocan’taddoracarpenterwhocan’tcutastraightlinewithasaw77hopelesscases.Thisbookhasbeendesignedtohelpyoucapitalize78thestrength

17andovercomethe79thatyoubringtothejoboflearning.Butforyourdevelopment,youmustfirst80stockofwhereyoustandnow.81wegetfurtheralonginthebook,we’llbe82insomedetailwithspecificprocessesfordevelopingandstrengthening83skills.However,84beginwith,youshouldpause85examineyourpresentstrengthsandweaknessesinthreeareasthatarecriticaltoyoursuccessorfailureinschool:your86,yourreadingandcommunicationskills,andyourstudyhabits.  67.[A]improvement[B]victory[C]failure[D]achievement  68.[A]a[B]the[C]some[D]certain  69.[A]in[B]on[C]of[D]to  70.[A]Outof[B]Of[C]To[D]Into  71.[A]who[B]what[C]how[D]which  72.[A]ensure[B]certain[C]sure[D]surely  73.[A]onto[B]on[C]off[D]in  74.[A]to[B]at[C]of[D]for  75.[A]near[B]on[C]by[D]at  76.[A]Have[B]Had[C]Having[D]Hadbeen  77.[A]being[B]been[C]are[D]is  78.[A]except[B]but[C]for[D]on  79.[A]idea[B]weakness[C]strength[D]advantage  80.[A]make[B]take[C]do[D]give  81.[A]As[B]Till[C]Over[D]Out  82.[A]deal[B]dealt[C]bedealt[D]dealing  83.[A]learnt[B]learned[C]learning[D]learn  84.[A]around[B]to[C]from[D]beside  85.[A]to[B]onto[C]into[D]with  86.[A]intelligence[B]work[C]attitude[D]weakness  PartⅥTranslation(5minutes)  Direction:CompletethesentencesonAnswerSheet2bytranslatingintoEnglishtheChinesegiveninbrackets.  87.IshouldsayHenryis______________(与其说是个作家不如说是)asareporter.  88.IntheChinesehousehold,grandparentsandotherrelatives______________(起着不可缺少的作用)inraisingchildren.  89.Mr.Johnsonmadefullpreparationfortheexperiment____________________(以便实验能顺利进行).  90.Pricesaregoinguprapidly.Petrolnow__________________________(价格是几年前的两倍).  91.Howcloseparentsaretotheirchildren__________________(有很强的影响)thecharacterofthechildren.【写作思路】  要求写一篇关于竞争的议论文。要求从以下几个方面阐述:竞争使人们充满创造力,使人们更有效率;竞争促使生产出更好的产品和提供更优质的服务;竞争促进了社会的进一步发展。

18  【参考范文】  Competition  Competitionmakespeoplemorecreativeandeffective.Iturgesthemtoplanbetter,totryharder,andtoachievemore.Withadesiretoexcelintheirwork,peopletendtoperformmorecreativelyandmoreefficiently.  Competitionhelpsproducebetterproductsandprovidebetterservices.Inthepresentworldofintensecompetition,everymanufacturetrieshisbesttooutmatchhiscompetitorbyproducinggoodsofbetterqualityatlowercosts.Asaresult,consumerspaylessandreceivebetterservice.  Competitionpromotesfurtherdevelopmentofsociety.Iftherewerenocompetition,peoplewouldremaincomplacentandbeunwillingtomakeanyeffortsorriskanythingnew.However,withfiercecompetitiongoingon,theyexertthemselvestoaccomplishmore.Theiraccomplishmentwill,inonewayoranother,contributetotheprogressofsociety.  PartIIReadingComprehension(SkimmingandScanning)  【全文翻译】  地震科学家能预测危险的地震吗?  1980年11月23日,意大利南部靠近那不勒斯的居民们感觉到大地晃动和颤抖。地震!建筑物在突然间倒塌了,大地出现了裂缝。几分钟内,整个城镇毁于一旦,成千上万的人死了,几千万的人受伤了。  当救援队在瓦砾堆中搜寻救人时,许多人想知道,如果受害者事先知道要发生地震的话,许多生命是否可以得到挽救。  事实上,意大利的一位科学家的确预测到了地震的发生。1977年,罗马DegilesStudi大学的博士M.Caputo先生警告说,不久那不勒斯东部将会发生大地震。不幸的是,他不能准确地预测地震发生的时间和日期。Caputo博士的预测是在与遍及意大利的54所地震监测站的科学家交谈后作出的。他发现,最近那不勒斯附近地区发生了多次地震。但是,多年来,那不勒斯东部的有个地方一直没有发生地震。他认为这个地区是在酝酿一次大地震。情况的确如此。  地震发生在空白区  Caputo博士把地震发生的区域称为地震空白区。地震空白区位于长期以来没有地震或者地震活动发生的地震活跃区。地震空白区位于地球两大板块的连接处。  当地震板块彼此滑过对方时,它们有时会被锁在一起。当你穿着运动鞋在人行道上跳跃时,也会发生类似的情况。当你双脚着地时,运动鞋会与粗糙的地面粘在一起,摩擦力会使你的脚后倾,同时你的身体却会前倾。你可能最终会跌得鼻青脸肿。  然而,地震板块表面是不均匀的,这会使它们锁在一起很多年。每个板块后都积聚着巨大的压力。当其中的一些能量释时,就会出现定期的震颤或者震动。  最后,在大约50年之后,地震空白区中的岩石或者突然崩裂或者以巨大的压力移动。这种突然释放的能量会让地震波穿过岩层直达地面,于是地面震动了,人行横道断裂了,建筑物倒塌了,强大的地震发生了。  利用地震空白区来预测地震  许多地质学家们已经使用地震空白区技术来准确地预测地震。该项技术是由苏联地震专家V.Fodotov博士在研究日本古代和近代地震时首先使用的。在标注日本已发生地震的地点规模和日期时,他发现了一个明显的特点。  日本所有的大地震都发生在几个孤立的地区。Fodotov博士指出,这些地方仅隔50到60年就会发生一次大地震;他还断言说,50年以上没有发生地震的区域是地震的“成熟”区。俄罗斯科学家将这些区域命名为地震空白区。

19  在过去的几年中,其他国家的地质学家在世界的其他地方也发现了地震空白区。经过对这些地区以往地震的详细研究,地质学家能够精确地预测地震发生的时间。  动物是如何知道地震即将来临的?  最近,试图预测地震的科学家们找到了一些新的帮手——动物。  没错,就是动物。科学家们开始了解到农民已经知道了数千年之久的情况。动物通常事先知道地震即将来临,并以奇怪的方式表现出恐惧。在中1975年国发生地震前,冬眠的蛇过早地醒来,结果冻死在寒冷的空气中;奶牛挣断缰绳,企图逃跑;鸡拒绝进入鸡舍。所有这些不寻常的行为以及地球自身的变化,使中国科学家充分地认识到地震即将来临。他们让地震危险区的居民搬迁出来,从而挽救了成千上万人的性命。  要准确了解到底哪类动物的行为可以预测地震,这是科学家目前的一个任务,但这不是一件容易的事情。首先不是每个动物都能对地震的危险作出反应。例如,在1977年加州地震之前,一匹阿拉伯种马变得非常紧张,试图跑出马厩;但是,他旁边的那匹马仍然非常安静。有时侯,也很难分辨出正常动物的不安和“地震(造成)的紧张”。有位动物管理员曾经对地震研究人员说,他的美洲狮一直行为怪怪的,结果证明是其胃部不舒服。  弄清楚到底动物收到了什么样的警告,这是科学家的另外一个任务。他们知道,动物的感觉比人的更敏感。许多动物能看到、听到、闻到人察觉不到的东西。一些动物能够感觉到气压、重力或者地球磁场的微小变化。这种特别的感觉可能有助于动物预测地震。  发生在一群狗身上的事情是这方面的一个很好的例子。关这群狗的地方正在发生一系列的小地震。(大地震前后常会发生一些小地震。)每次地震前都能听到一声低沉的爆炸声,每次爆炸声都会引起狗的疯狂乱叫。后来,狗在没有爆炸声一段时期也开始叫喊。一位正在记录瓦块地震的科学家注意到他的机器也好像表现出有巨大声音的样子。他意识到,狗对一次巨大的声音作出了反应,它们也感觉到了随后发生的轻微地震。尽管人类什么都没有感觉到、也没有听到,这台机器却记录下了这一切。  在这种情况下,有一个机器监视着狗的感应。然而,有许多次,尽管动物知道地震即将来临,但我们的机器却没有记录下任何异常的活动。动物可能会感觉到我们所测量的情况,但没有把它看作是一种警告。发现动物感应到什么,并了解他们是如何知道这是一种危险的信号,这是科学家今后的工作。  【答案解析】  1.【解析】[Y]根据第一段。  2.【解析】[Y]根据earthquakesstrikesingap的第一段判断。  3.【解析】[Y]根据gapsusedtopredictquakes中的最后一段最后一句判断正确。  4.【解析】[N]根据第十二段,Cowsbroketheirhaltersandtriedtoescape.并没有交代ranawayfromtheirsheds。  5.【解析】[N]这里的cat指的是cougar。  6.【解析】[N]根据第十四段最后三句话,我们可以得出,并不是所有的动物都能感受到的。  7.【解析】[Y]参看第十五段。  8.【解析】east根据第三段,ButnonehadoccurredinoneparticularspoteastofNaplesformanyyears.  9.【解析】theseismicgap根据第七段,Finally,afterabout50years,rockintheseismicgapeithersuddenlybreaksormovesunderthegreatstress.  10.【解析】thedangerzone/theprospectivequakezone根据第十四段,Theymovedpeopleawayfromthedangerzoneandsavedthousandsoflives.如果能写出theprospectivequakezone也是可以的。

20PartⅣReadingComprehension(ReadinginDepth)  SectionA  【全文翻译】  美国人如何看待食物呢?我们爱吃,但是过后会觉得内疚。我们说我们只要最好的,但是却热衷于享受垃圾食品;我们很重视健康和减肥,但是肥胖却又空前地在蔓延。也许,这种矛盾源于我们的历史。首批来到这个大陆的欧洲人是为了寻找新的香料,但却一无所获;首种经济作物不是用来吃的,而是用来吸得。那时,旨在禁止酗酒的禁酒令,实际上却激发了更多新奇的方法来酗酒。  移民的经历也与此有内在的关系。入乡随俗意味着要吃“典型的美国人”的饮食,但是美国的食物已经被诸如比萨或者热狗这样的舶来品所诠释了。美国最珍贵的一些烹饪来自于戴着镣铐而来的人们。  无论是在波士顿倾茶事件中,还是在南部午餐柜台前的静坐中,食物都曾被用来作为界定斗争的一种手段,这也许是不足为奇的。无论是出于宗教原因而戒酒,还是由于政治避难而戒肉,这都与我们的健康甚至是道德的观念相一致。  但是,这些坚定的观点并不是确定不变的。美国人对他们所吃的食物的态度是矛盾的。我们怀疑我们的食物,特别是当我们更清楚地了解了它们的成分时。  对食物的信仰一直存在于美国人的意识之中。第一个感恩节就把美国人的想象力束缚在食物之中,束缚在我们与朋友对它的分享之中,这并不是偶然的。  【答案解析】  47.【解析】[D]feel是一个系动词,可以判断此处应填入一个形容词,通过上下文意思,以及后面介词about,可以确定选项为D项guilty,短语feelguiltyaboutsth.“对……感到有愧”。全句的意思为“我们很爱吃,但是往往在吃完之后又有负罪感”。  48.【解析】[M]beobsessedwith为固定搭配,原意为“被……附上/缠住/迷住心窍”,放在本句表示“十分重视”。全句的意思为“我们很重视健康和减肥,但肥胖却又空前地在蔓延”。  49.【解析】[A]本句缺一个名词作主语,并且根据和介词to的搭配,可以推断出正确选项answer。  50.【解析】[I]本句根据more和ways可以判断出需要填入一个形容词构成比较级,根据上下文,表示“旨在禁止酗酒的禁酒令,却激发了更多新奇的方法来酗酒”,可以确定I为正确选项。  51.【解析】[F]本题较难。根据be和by确定应填入一个过去分词。再根据上下文,上文表示“应该吃典型的美国人吃的食物”,下文通过but转折,表示实际上“美国的食物已经被诸如比萨和热狗这样的舶来品所诠释了”,因此可以确定F为正确选项。  52.【解析】[B]政治结果,可根据宗教原因religiousreasons来推断此处填政治结果。  53.【解析】[L]由于横线后面没有宾语,可以确定不是形成bring的短语,这样本句所缺的为一名词,作bring的宾语。根据下文解释,“美国人对他们所吃的食物的态度是矛盾的”,可以推出本句意思为“坚定的观点也不是确定不变的”。因此可以确定L为正确选项。  54.【解析】[K]系动词become后应填入一个形容词,和后面介词of形成短语be/becomesuspiciousof“对……感到怀疑”。  55.【解析】[J]本句缺一个名词作主语,并且根据和介词in的搭配,可以推断出正确选项belief,(have)beliefinsth.“相信……”。  56.【解析】[C]本句是一般现在时,缺一个动词,且和with搭配,确定选项share,sharesth.withsb.,“与某人分享某事”。  SectionB

21  PassageOne  【全文翻译】  欧洲南部的妇女在10和11世纪时享有较高的社会地位,这并未获得广泛认识。作为一个妻子,女性的地位受到其嫁妆的保护。诚然,嫁妆最初的目的是防止女性被抛弃;但是,它在当时家庭和社会现实生活中起着更重要的作用。妻子的嫁妆使她有权获得其丈夫十分之一的财物。妻子有权利拒绝丈夫所做的任何交易,但这不仅仅只是一项权利而已;文件表明她与丈夫一样平等地享有真正的决定权。文件没有表明丈夫和妻子在法律地位上有任何差别。  妻子享有管理丈夫私人财产的权利,但是反之则不然。如果丈夫要越权侵犯她们的利益,女性们会时刻准备着捍卫自己的利益,有时她们还会表现出一种坚强的斗争精神。MariaVivas就是一个典型的例子。为了家庭的需要,她同意其丈夫Miro出售一块属于她的土地,但是她坚持要求获得补偿。但是丈夫没有给她提供补偿,于是她把丈夫拖到一个文书处,起草了一份合同,成功地把他丈夫的一块私人土地划归自己。正如合同所写的,“为了和平”,这个不幸的丈夫不得不同意。要么借助嫁妆,要么通过发脾气,妻子知道如何在家庭中为自己赢得强大的经济地位。  57.【解析】[C]推断题。本文介绍了欧洲南部10和11世纪嫁妆对女性在婚姻中地位的重要性。虽然嫁妆最初的目的是防止女性被抛弃,但实际上它的作用远远不止于此。它使妻子在婚姻中的地位与丈夫平等,并保障了妻子的经济利益。女性在维护自己的利益上,是很坚定果敢的,MariaVivas就是一个例子。见第一段第三句,“Admittedly,thepurposeofthiswastoprotectheragainsttheriskofdesertion...”,虽然在实际生活中嫁妆有更重要的作用,但最初它的作用只是为了防范女性被丈夫抛弃,所以C为正确答案。  58.【解析】[D]细节题。根据本文,妻子在婚姻中的法律地位和丈夫是平等的。见第一段最后一句,“Innocasedothedocumentsindicateanydegreeofdifferenceinthelegalstatusofhusbandandwife”文件中并没有表明丈夫和妻子在法律地位上有任何的差别,也就是说他们的地位是平等的,所以D为正确答案。  59.【解析】[B]推断题。作者举MariaVivas的事例,是为了说明妻子是能够捍卫自己的利益的。见第二段第二句,“Womenseemedperfectlypreparedtodefendtheirowninheritance...theyshowedafinefightingspirit.”如果丈夫要侵占自己的利益,女性们就时刻准备着捍卫自己的利益,有时她们还表现出很强的斗争精神。接下来作者就举了MariaVivas的例子,说明女性是有能力捍卫自己的利益的。所以B为正确答案。  60.【解析】[A]细节题。MariaVivas得到的一份作为补偿的土地,本来是属于Miro的个人财产。见第二段第五句,“Nonebeingoffered,shesucceededin...assigningherapieceoflandfromMirospersonalinheritance”由此知A为正确答案。  61.【解析】[D]推断题。作者对MariaVivas所持的态度是客观的。作者客观地介绍了MariaVivas的事例,并没有表示出同情、不满或者是漠不关心,所以A、B、C都是错误答案。PassageTwo  【全文翻译】  社会学家认为,有几种不同的方法可以使人成为某个社会群体公认的领导者。在家庭生活中,传统的文化模式赋予父母一方或者双方领导的地位。在其他情况下,比如友谊团体,尽管没有正式的选举程序,一个或更多的人会逐渐地成为领导。在比较大的群体中,领导者通常是通过正式的选举或招募的形式产生的。  虽然通常认为领导具有人不同寻常的个人能力,数十年的研究未能找出可靠的证据来证明那些人是“天生的领导者”。看起来似乎没有所有的领导都具有的固定的个人品格;相反,只要具有适合某个特别群体所需要的品质,几乎任何人都可以成为其领导。

22  研究表明,通常不同的人会充当两种不同的领导角色。指导型领导注重社会群体目标的实现。群体成员期望指导型领导人“把事情搞定”。另一方面,表现型领导则强调社会群体成员的集体幸福。表现型领导并不侧重于群体目标的实现,而是注重为群体成员提供情感支持,并尽力减少内部的紧张和纷争。  指导型领导与其他的群体成员之间的关系可能是相当次要的。他们可能会约束阻止实现群体目标的成员的行为。表现型领导则与群体其他成员之间建立了一种更私人的或者相当重要的关系。当有人经历过困难时,他们表示同情,并设法解决危及群体分裂的问题。正如这两种不同角色所表明的那样:表现型领导一般会从群体成员那里获得更多的个人感情;而指导型领导,如果他们能成功地实现群体的目标,可能会获得更广泛的尊重。  【答案解析】  62.【解析】[D]主旨题。本篇主要讨论不同的领导者(如instrumentalleaders以及expressiveleaders)在不同的社会群体中所充当的角色和发挥的功能。所以D为正确答案。A、B、C选项都不能正确概括本文主旨。  63.【解析】[C]细节题。A选项的招募(recruitment)和B选项选举程序(electionprocess)都在第一段中作为大群体的领导者产生的方式被明确提到。D选项也在第一段中作为家庭领导者的产生的方式被明确提到。只有C选项没有作为产生领导者的一种方式在本文中被提及。  64.【解析】[A]推断题。A选项意思是:某一特定群体的有效领导者,不一定就可以成为另一群体的有效领导者。这正好与第二段的意思相吻合,即领导者没有一个固定的特点,只是符合了一个特定群体需要的人就有可能成为该群体的领导者(Itseems...virtuallyanypersonmayberecognizedasaleaderifthepersonhasqualitiesthatmeettheneedsofthatparticulargroup)。B选项和C选项的意思与本文无关。D选项意为:许多人想成为领导者,但却很少能拿出证据来证明他们具有这样的资格。显然是对文章意思的曲解。  65.【解析】[B]细节题。第二段中“...decadesofresearchhavefailedtoproduceconsistentevidencethatthereisanycategoryof‘naturalleaders’”意思是说数十年来的研究并不能找出可靠证据来证明哪些人可以成为“天生领导者”,与B项意思一致。  66.【解析】[D]细节题。第三、四段主要讲了两种类型的领导者的区别。其中instrumentalleader侧重于群体目标的实现,而expressiveleader相对于前者并不侧重于群体目标的实现,而是注重为群体成员提供情感支持,并尽力减少内部纷争。(Expressiveleadersarelessconcernedwiththeoverallgoalsofthegroupthanwithprovidingemotionalsupporttogroupmembersandattemptingtominimizetensionandconflictamongthem.)  PartⅤCloze  【全文翻译】  如果你计划明天开始一个新的工作,你的一些基本的优缺点也会与你同行。成功或失败在很大的程度上取决于你是否能够扬长避短。最重要的是你的态度。如果一个刚刚开始工作的人就深信自己不会喜欢或者肯定不适应这项工作,那么妨碍他成功的缺点就暴露出来了。另一方面,如果一个人确信他可能会与其他人一样有能力完成这项工作,并心甘情愿地去尝试,他就具有了某种目的性的优势。很可能他会做得很好。具有某一工作的必要技能是一种优势,而缺乏这些技能显然就会处于劣势。一个记账员不会计数,或者一个木匠不能用锯子切割出一条直线,这都是令人绝望的例子。本书的目的是帮助你发挥自己的优势,克服自己学习中的劣势。但是为了你的发展,你必须要清楚自己现在的位置。随着书中内容的进一步深入,我们将详细论述发展和加强学习技能的具体过程。但是,首先,你应该停下来,去审视一下自己目前在以下三个方面存在的优点和缺点,这三个方面对你学业的成功或失败至关重要:态度、阅读和沟通的技巧、学习习惯。  【答案解析】

23  67.【解析】[C]本句的意思是“成功或失败在很大程度上取决于你是否能扬长避短。”improvement改进;victory胜利;achievement成果,成就。这三个词都不合题意,只有[C]failure“失败”最合适。  68.【解析】[A]toagreatextent是固定短语,意思是“很大程度上”,符合题意。类似的说法还有:toalargeextent,tosomeextent,toanextent,toacertainextent,tothatextent,totheextentof...。the,some,certain都不能与greatextent搭配。  69.【解析】[B]on与前面的depend构成本句谓语。  70.【解析】[B]of与名词连用,表示具有某种性质、状态,作表语。置于句首,表示强调。正常语序为:Yourattitudeisoftheutmostimportance.  71.【解析】[A]person后应当是定语从句,“开始工作的那个人”。  72.【解析】[C]本句的意思是“如果一个刚刚开始工作的人就深信自己不会喜欢或肯定不适应这项工作,那么阻碍他成功的缺点就暴露出来了。”or后面省略了主语he。ensure保证;certain肯定的(只能用于it作主语的句子里);surely确实地;sure肯定的。  73.【解析】[D]inone’sbelief相信。其他选项都不能与belief搭配。  74.【解析】[C]capableofdoing是固定搭配,意为“能够干什么”。  75.【解析】[D]attempt用作名词,后接介词at,意为“试图,努力”;如果后面接介词on,表示攻击的意思。其他两项都不能与attempt搭配。  76.【解析】[C]本句的意思是“具有某一工作的必要技能是一个优势”,此处应填句子的主语,故选动名词having。  77.【解析】[D]本句主语是Abookkeeperorcarpenter...。根据主谓一致原则,其谓语应当用单数is,而不是复数are。being,been都是分词,应该排除。  78.【解析】[D]on与前面的capitalize搭配,表示“利用”。作不定式help的补语。其他选项不能与capitalize搭配。  79.【解析】[B]本句的意思是“克服缺点”,应选weakness(缺点,弱点)。idea观点;strength优点;advantage优势。  80.【解析】[B]固定短语takestockof,意为“对……估价,对……作出判断”。  81.【解析】[A]本句的意思是:“随着更深入的阅读”,从语法角度,此处应填关联词。四个选项中,只有as能用作关联词。  82.【解析】[D]选项A、B、C分别是deal(处理,论述,涉及)的原形、过去分词及被动语态。根据语法和语义,此处应填将来进行时,意思是“随着书中内容的进一步深入,我们将详细论述发展和加强学习技能的具体过程。”  83.【解析】[C]根据上下文,这里应当指“学习技能”。  84.【解析】[B]固定短语tobeginwith,意为“首先,第一”,常用作插入语。  85.【解析】[A]谓语动词pause后,examine又是动词原形,所以这里应当选to,可构成动词不定式,作目的状语。其余选项都是介词,不合题意。  86.【解析】[C]本句对全文进行总结概括,与文章的第三句相呼应,Oftheutmostimportanceisyourattitude故选attitude。  PartⅥTranslation  87.【答案】notsomuchawriter  88.【答案】playindispensableroles  89.【答案】sothatitcouldbeconductedsmoothly  90.【答案】istwiceasexpensiveasitwasafewyearsago  91.【答案】hasastronginfluenceon2012年6月英语四级模拟题三

24四级作文:  Directions:Forthispart,youareallowed30minutestowriteABriefIntroductiontotheUniversity.Youshouldwriteatleast120wordsaccordingtothefollowingguidelines:  假设你是一名学生志愿者,要给来你们学校参观的外国朋友介绍学校情况。内容应包括学校历史、规模、教学现状以及未来目标等。  ABriefIntroductiontotheUniversity  Distinguishedguests,  Welcometoouruniversity.Beforeyoustarttolookaround,allowmetogiveyouabriefaccountoftheschool.  Foundedin1927,ouruniversityisoneofthiscity’searliestuniversitiesofliberalarts.Itisstaffedwithanexcellentfaculty,andhasatotalenrollmentofover10,000students.Inthepastyears,ithasturnedoutnumerouswell-qualifiedstudentsandfounditsgraduatesactiveinprofessionsofallwalksoflife.  Sinceitsestablishment,theuniversityhasalwayssteereditselftowardtheobjectivethatitsstudentshaveanoverallhealthydevelopment.Notonlydoesitprovidethestudentswithbasicacademiccourses,butitmanagestoexposethemtotheup-to-dateknowledge.Besides,studentsarefreetoparticipateincolorfulcampusactivitiesandsocialpractice,whichareintendedforbroadeningtheirmindanddevelopingtheirpotentialtalent.  Currently,bothourfacultyandstudentsaremakingeveryefforttoimprovethequalityofoureducationinthedirectionofafirst-rateuniversity.Thankyou.  点评:近几年,随着我国顺利加入世贸组织以及经济全球化步伐的加快,许多国内高校纷纷与国外大学合作办学,教育走向国际化成为人们关注的热点,也是大学生门比较关心的热点问题。本预测题为热议校园生活话题,与2004年旅游景点介绍2006年考查的名校校园开放如出一辙。四级阅读:BaekelandandHartmannreportthatthe“shortsleepers”hadbeenmoreorlessaverageintheirsleepneedsuntilthemenwereintheirteens.Butataboutage15orso,themenvoluntarilybegancuttingdowntheirnightlysleeptimebecauseofpressuresfromschool,work,andotheractivities.Thesementendedtoviewtheirnightlyperiodsofunconsciousnessasbothersomeinterruptionsintheirdailyroutines.  Ingeneral,these“shortsleeps”appearedambitious,active,energetic,cheerful,conformist(不动摇)intheiropinions,andverysureabouttheircareerchoices.Theyoftenheldseveraljobsatonce,orworkersfull-orpart-timewhilegoingtoschool.Andmanyofthemhadastrongurgetoappear“normal”or“acceptable”totheirfriendsandassociates.  Whenaskedtorecalltheirdreams,the“shortsleepers”didpoorly.Morethanthis,theyseemedtoprefernotremembering.Insimilarfashion,theirusualwayofdealingwithpsychologicalproblemswastodenythattheproblemexisted,andthentokeepbusyinthehopethatthetroublewouldgoaway.  Thesleeppatternsofthe“shortsleepers”weresimilarto,butlessextremethan,sleeppatternsshownbymanymentalpatientscategorizedasmanic(疯人).  The“longsleepers”werequitedifferentindeed.BaekelandandHartmannreportthattheseyoungmenhadbeenlengthysleepssincechildhood.Theyseemedtoenjoytheirsleep,protectedit,andwerequiteconcernedwhentheywereoccasionallydeprivedoftheirdesired9hoursofnightlybedrest.Theytendedtorecalltheirdreamsmuchbetterthandidthe“shortsleepers.”

25  Manyofthe“longsleepers”wereshy,anxious,introverted(内向),inhibited(压抑),passive,mildlydepressed,andunsureofthemselves(particularlyinsocialsituations).Severalopenlystatesthatsleepwasanescapefromtheirdailyproblems.  1.Accordingtothereport,______.  A)manyshortsleepersneedlesssleepbynature  B)manyshortsleepersareobligedtoreducetheirnightlysleeptimebecausetheyarebusywiththeirwork  C)longsleeperssleepalongerperiodoftimeduringtheday  D)manylongsleeperspreservetheirsleepinghabitformedduringtheirchildhood  2.Many“shortsleepers”arelikelytoholdtheviewthat_____.  A)sleepisawithdrawalfromthereality  B)sleepinterfereswiththeirsoundjudgement  C)sleepistheleastexpensiveitemontheirroutineprogram  D)sleepisthebestwaytodealwithpsychologicaltroubles  3.Itisstatedinthethirdparagraphthatshortsleepers_____.  A)areideallyvigorousevenunderthepressuresoflife  B)oftenneglecttheconsequencesofinadequatesleep  C)donotknowhowtorelaxproperly  D)aremoreunlikelytorunintomentalproblems  4.Whensometimestheycannotenjoyadequatesleep,thelongsleepersmight____.  A)appeardisturbed  B)becomeenergetic  C)feeldissatisfied您看到来自www.233.com的新闻  D)beextremelydepressed  5.WhichofthefollowingisNotincludedinthepassage?  A)Ifonesleepsinadequately,hisperformancesuffersandhismemoryisweakened  B)Thesleeppatternsofshortsleepersareexactlythesaneasthoseshownbymanymentalpatients  C)Longandshortsleepersdifferintheirattitudestowardssleep  D)Shortsleeperswouldbebetteroffwithmorerest  【答案及详解】  答案:DCBAB  贝克尔和哈特曼报道说,“睡眠少的人”在未进入少年期之前,其正常睡眠时间大致与所需要的时间差不多。但到了15岁左右,由于学校、工作或其它活动的地压力,他们就故意地减少了夜间睡眠的时间。这些人持有这样的观点:夜间睡眠是一件令人讨厌的事情,打断了日常事务。  总的说来,这些“睡眠少的人”表现得雄心勃勃、积极活跃、精力充沛、无意识乐观豁达、立场坚定,对自己职业的选择胸有成竹。他们往往同时从事几项工作,或者一边上学读书,一边从事专职或兼职工作。其中许多人有强烈愿望,想在朋友和熟人面前表现得“正常”或“合群”。当让他们回忆梦境时,“睡眠少的人”回忆不起什么来。更有甚者,他们似乎情愿什么都记不住。类似的情况是他们通常处理心理问题的方式:不承认问题的存在,希望只要忙忙碌碌,麻烦总会过去的。  “睡眠少的人”

26的睡眠模式与被划入疯子之类精神病患者的睡眠模式十分相似,只不过没有那么严重而已。  “睡眠多的人”情形则大不相同。贝克尔和哈特曼报道说,这些年轻人从小的,有抱负的睡眠就一直很长。他们好像注重睡眠,不让睡眠受打搅。偶尔没有所需的9个小时夜间卧床休息,他们便会十分不安。他们比“睡眠少的人”要更能回忆得起梦的内容。许多“睡眠多的人”腼腆、焦躁、内向、压抑、消极和稍微有点儿沮丧,尤其在社交场合缺乏自信。好几个人坦言,睡眠是摆脱每天烦恼的一种方式。  Nowletuslookathowweread.Whenwereadaprintedtext,oureyesmoveacrossapageinshort,jerkymovement.Werecognizewordsusuallywhenoureyesarestillwhentheyfixate.Eachtimetheyfixate,weseeagroupofwords.Thisisknownastherecognitionspanorthevisualspan.Thelengthoftimeofrwhichtheeyesstop---thedurationofthefixation----variesconsiderablyfrompersontoperson.Italsovaieswithinanyonepersonaccordingtohispurposeinreadingandhisfamiliaritywiththetext.Furthermore,itcanbeaffectedbysuchfactorsaslightingandtiredness.  Unfortunately,inthepast,manyreadingimprovementcourseshaveconcentratedtoomuchonhowoureyesmoveacrosstheprintedpage.Asaresultofthismisleadingemphasisonthepurelyvisualaspectsofreading,numerousexerciseshavebeendevisedtotraintheeyestoseemorewordsatonefixation.Forinstance,insomeexercises,wordsareflashedontoascreenfor,say,atenthoratwentiethofasecond.Oneoftheexerciseshasrequiredstudentstofixtheireyesonsomecentralpoint,takinginthewordsoneitherside.Suchwordpatternsareoftenconstructedintheshapeofrathersteeppyramidssothereadertakesinmoreandmorewordsateachsuccessivefixation.Alltheseexercisesareveryclever,butit’sonethingtoimproveaperson’sabilitytoseewordsandquiteanotherthingtoimprovehisabilitytoreadatextefficiently.Readingrequirestheabilitytounderstandtherelationshipbetweenwords.Consequently,forthesereasons,manyexpertshavenowbeguntoquestiontheusefulnessofeyetraining,especiallysinceanyapproachwhichtrainsapersontoreadisolatedwordsandphraseswouldseemunlikelytohelphiminreadingacontinuoustext.  Q:  1.Thetimeoftherecognitionspancanbeaffectedbythefollowingfactsexcept________.  A.one’sfamiliaritywiththetext  B.one’spurposeinreading  C.thelengthofagroupofwords  D.lightingandtiredness  2.Theauthormaybelievethatreading______.  A.requiresareadertotakeinmorewordsateachfixation  B.requiresareadertoseewordsmorequickly  C.demandsandeeply-participatingmind  D.demandsmoremindthaneyes  3Whatdoestheauthormeanbysaying“butit’sonethingtoimproveaperson’sabilitytoseewordsandquiteanotherthingtoimprovehisabilitytoreadatextefficiently.”inthesecondparapraph?  A.Theabilitytoseewordsisnotneededwhenanefficientreadingisconducted.  B.Thereadingexercisesmentionedcan’thelptoimproveboththeabilitytoseeandtocomprehendwords.  C.Thereadingexercisesmentionedcan’thelptoimproveanefficientreading.

27  D.Thereadingexercisesmentionedhasdoneagreatjobtoimproveone’sabilitytoseewords.  4.WhichofthefollowingisNOTtrue?  A.Thevisualspanisawordoragroupofwordsweseeeachtime.  B.Manyexpertsbegantoquestiontheefficiencyofeyetraining.  C.Theemphasisonthepurelyvisualaspectsismisleading.  D.Theeyetrainingwillhelpreadersinreadingacontinuoustext.  5.Thetuneoftheauthorinwritingthisarticleis________  Acritical  Bneutral  Cprssimistic  Doptimistic  【答案及详解】  答案:CCCDA  解题思路  1C。事实细节题。第一段提到了影响视幅的因素:不同的人,不同的阅读目的,对材料的熟悉程度,光线,疲劳。C“一组词的长度”不是能影响视幅的因素,是本题的答案。  2C。观点态度题。作者在第二段第二句提到,眼睛训练课程只注重了阅读的视觉因素。倒数第二句书哦,阅读要求具备理解单词间关系的能力。因此选项C“阅读需要大脑的深度参与”正确。作者没有否定阅读的视觉因素的必要性,但是也没有说读者应该练习拓宽视幅,加快阅读速度。所以AB两项都不正确。作者没有对大脑和眼睛在阅读过程中的重要性进行对比,所以不选D  3C。作者在这句话中先是肯定了那些阅读练习提高看单词的能力,然后提出了有效阅读概念。后面句中还指出,有效阅读需要的是理解单词间的联系的能力。所以作者的意思应该是那些(训练眼睛的)阅读练习对与有效的阅读无益。B与文章相反。D句是蕴涵其中一个意思,却不是作者想表达的方向。  4D。第二段最后一句说,眼睛训练对于帮助读者阅读连贯文章无益,所以D是错的,是本题答案。  5A。参考前面的结构剖析,作者写本文的主要目的是对那些只关注阅读的视觉因素的阅读能力课程进行批判,所以答案应该是A。  Asisknowntoall,theorganizationandmanagementofwagesandsalariesareverycomplex.Generallyspeaking,theAccountsDepartmentis__1__forcalculationsofpay,whilethePersonnelDepartmentisinterestedindiscussionswiththeemployeesaboutpay.  Ifafirmwantsto__2__anewwageandsalarystructure,itisessentialthatthefirmshoulddecideona__3__ofjobevaluationandwaysofmeasuringtheperformanceofitsemployees.Inordertobe__4__,thatnewpaystructurewillneedagreementbetweenTradeUnionsandemployers.Injobevaluation,alloftherequirementsofeachjobaredefinedinadetailedjobdescription.Eachofthsoerequirementsisgivenavalue,usuallyin"points",whichare__5__togethertogiveatotalvalueforthejob.Formiddleandhighermanagement,aspecialmethodisusedtoevaluatemanagersontheirknowledgeofthejob,theirresponsibility,andtheir__6__tosolveproblems.Becauseofthedifficultyinmeasuringmanagementwork,however,jobgradesformanagersareoftendecidedwithout__7__toanevaluationsystembasedonpoints.  Inattemptingtodesignapaysystem,thePersonnelDepartmentshould__8__thevalueofeachjobwiththeseinthejobmarket.__9__,paymentforajobshouldvarywithanydifferences

28inthewaythatthejobisperformed.Whereitissimpletomeasuretheworkdone,asintheworksdonewithhands,monetaryencouragementschemesareoftenchosen,for__10__workers,wheremeasurementisdifficult,methodsofadditionalpaymentsareemployed.  [A]compare  [B]responsible  [C]useful  [D]added  [E]find  [F]reference  [G]indirect  [H]method  [I]successful  [J]combined  [K]Necessarily  [L]capacity  [M]ability  [N]Basically  [O]adopt  【答案及详解】  1.选B)。此处应填形容词。原文意思为“会计部门...计算报酬”,选项中的形容词responsible“负责任的”,useful“有用的”,indirect“间接的”,successful“成功的”,其中indirect不能与for连用,排除;剩余几项中只有B)responsible意义符合原句,其他均不符合,故排除。  2.选O)。此处应填动词原形。本文主题就是采用一种新的工资和薪水制度时需要注意的问题,选项中的动词原形有compare“比较”,find“找到”,adopt“采纳”,分别带入原文,只有adopt最符合原文意思,故选O).  3.选H)。此处应填名词。从原文看,and连接并列结构,所以要填的词应与and后面的ways意思一致,选项中的名词只有mathod=ways,故选H)method。  4.选I)。此处应填形容词。上文说新的工资制度需要一套决定工作评估和衡量雇员表现的方法,说的是制度“是否有用”的问题。这句说的时新的工资制度执行过程中的问题,劳资双方先期达成一致是薪酬制度成功的必要条件。形容词useful和successful,I)seccessful更符合原文意思。  5.选D)。此处应填动词。这个动词的宾语是point“分值”,把分值...起来togiveatotalvalue"得出总分值",选项中有added和combined,前者指“把...相加”,后者意为“把...结合在一起”,原文指将分值相加得出总分,故D)added最符合文意。  6.选M)。此处应填名词。首先solveproblem“解决问题”是经理们应具备的能力,选项中的capacity与ability都可以表示“能力”;前者强调的是理解的能力和接受事物的能力,而后者强调实际应用的能力,故不难判断解决问题的能力应该用M)ability。  7.选F)。此处应填名词。前半句指出“因为管理工作很难评估,经理的工作得分不需要参照基于分值的评估系统决定。”withoutreferenceto为固定搭配,意思是“与...无关”。故选F)reference.  8.选A)。此处应填动词原形。原句中出现了thevalueofeachjob“每种工作的价值”和theseinthejobmarket“工作市场上的(工作价值)”,说明人事部门通过比较两种价值来计算工资制度。选项中只有compare表示“比较”,所以A)正确。

29  9.选K)。此处应填副词,修饰整句话。“报酬要随工作表现各方面的不同而有所改变”,选项中Necessarily“必须地”与Basically“基本地”为副词,分别带入原文,“报酬...的变化是必须的”更符合上下文意思,故排除N),选K)。  10.选G)。此处应填形容词。前文中提到一种情况即simpletomeasuretheworkdone"易于评估的工作",通常用现金奖励办法;而...measurementisdifficult“难于评估的”,说明这些工作人员的工作不像手工工作那么直接,因而是间接的,故选项中只有G)indirect符合原句的意思。  Americansareproudoftheirvarietyandindividualty,yettheyloveandrespectfewthingsmorethanauniform.Whyareuniformsso__1__intheUnitedStates?  Amongtheargumentsforuniforms,oneofthefirstisthatintheeyesofmostpeopletheylookmore__2__thancivilian(百姓的)clothes.Peoplehavebecomeconditionedto__3__superiorqualityfromamanwhowearsauniform.Thetelevisionrepairmanwhowearsauniformtendsto__4__moretrustthanonewhoappearsincivilianclothes.Faithinthe__5__ofagaragemechanicisincreasedbyauniform.Whataneasierwayisthereforanurse,apoliceman,abarber,orawaiterto__6__professionalidentity(身份)thantostepoutofuniform?Uniformsalsohavemany__7__benefits.Theysaveonotherclothes.Theysaveonlaundrybills.Theyareoftenmorecomfortableandmoredurablethancivilianclothes.  Primaryamongtheargumentsagainstuniformsistheirlackofvarietyandtheconsequentlossof__8__experiencedbypeoplewhomustwearthem.Thoughtherearemanytypesofuniforms,thewearerofanyparticulartypeisgenerallystuckwithit,without__9__,untilretirement.Whenpeoplelookalike,theytendtothink,speak,andact__10__,onthejobatleast.  [A]skill  [B]popular  [C]get  [D]change  [E]similarly  [F]professional  [G]character  [H]individuality  [I]inspire  [J]differently  [K]expect  [L]practical  [M]recall  [N]lose  [O]ordinary  【答案及详解】  1.选B)。从文章的第一句theyloveandrespectfewthinsmorethanauniform“他们又无比热爱和崇尚制服”,说明了制服在美国很受欢迎。因此选项B)popular符合原文意思。选项中的professional“职业的”、practical“实用的”和ordinary“普通的,平常的”都不符合第一段的意思。  2.选F)。此处应填形容词。从more...thancivilianclothes可知,此处要填的形容词意思与civilian相对,说明制服的特点。选项中的形容词中只有professional“职业的”和civilian相对的,故F)正确。而pratical“实用的”和ordinary“平常的”

30意思都不能和civilian对应,故排除。  3.选K)。由beconditionedtodosth.“习惯于”可知,此处应填动词原形。从前面的lookmore“看起来更...”和后面的tendto“倾向于”可知,这段要说明的是人们的主观印象,应填入表示“期望(得到)”的单词,只有K)expect符合原文语气。选项中get“得到”与上下文的语气不符合。  4.选I)。此处应填动词原形。从原文中“人们习惯...从穿制服的人那儿得到优质服务。”可知,人们更信任穿制服的人,即制服能使人产生信任感。选项中的动词原形中只有inspire“使产生”符合文章。  5.选A)。此处应填名词。前面两句表达了人们对穿制服的人更加信任的意思,那么对于garagemechanic“汽车修理工”来说,人们信任的是它的技术,而不是人品,故选项中只有A)skill符合原文意思。  6.选N)。此处应填动词。...stepoutofuniform“脱掉制服”是对护士、警察等来说是......职业身份的很简单的方式。脱下制服就是失去了职业身份,由此可以推知此处应填N)lose。  7.选L)。此处应填形容词,说明制服的其他有点。从下文“...节省购买其他衣服的开销,节省洗衣费用,比便服更舒适也更耐穿”可知,制服除了增加信任感还有实际的优点。选项中的pratical“实用的”和ordinary“平常的”,很明显L)pratical符合原文意思。  8.选H)。此处应填名词。文章手段就说“美国人为自己的多元化和个性化感到高傲无比,然而他们又无比热爱和崇尚制服”,其中包含制服使他们失去自己的个性的意思,那么联系第一段,此处指出的制服的缺点即为失去个性,故选项H)individuality正确。individuality强调与他人特点的区别,而character指的是个人特定的内在本质。  9.选D)。此处应填名词。前面说制服让人失去了个性,虽然有很多种制服,但穿上制服的人直至退休都是那件制服,所有是没有变化的,故此处应填change,故D)正确。  10.选E)。此处应填副词,修饰动词act。前面指出...lookalike,theytendto...,说明此处填的词和alike意思相近。选项种副词有similarly和differently,很明显,E)similarly与alike意思相近,  AftertheviolentearthquakethatshookLosAngelesin1994,earthquakescientistshadgoodnewstoreport:Thedamageanddeathtoll(死亡人数)couldhavebeenmuchworse.Morethan60peoplediedinthisearthquake.Bycomparison,andearthquakeofsimilar__1__thatshookAmericain1998claimed25,000victims.  Injuriesanddeathswere__2__lessinLosAngelesbecausethequakeoccurredat4:31a.m.Onaholiday,whentrafficwaslightonthecity’shighway.Inaddition,__3__madetotheconstructioncodesinLosAngelesduringthelast20yearshavestrengthenedthecity’sbuildingsandhighways,makingthemmore__4__toquakes.  Inthepast,makingstructuresquake-resist-antmeantfirmyet__5__materials,suchassteelandwood,thatbendwithoutbreaking.Later,peopletriedtoliftabuildingoffitsfoundation,andinsertrubberandsteelbetweenthebuildinganditsfoundationto__6__theimpactofgroundvibrations.Themost__7__designsgivebuildingsbrainsaswellasconcreteandsteelsupports,calledsmartbuildings,thestructuresrespondlikelivingorganismstoanearthquake’svibrations.Whengroundshakesandthebuildingtipsforward,thecomputerwould__8__thebuildingtoshiftintheoppositedirection.Thenewdesignsshouldofferevengreater__9__tocitieswhereearthquakesofentakeplace.  Thenewsmartstructurescouldbevery__10__tobuild.However,theywouldsavemanylivesandwouldbelesslikelytobedamagedduringearthquakes.  [A]changes

31  [B]flexible  [C]decrease  [D]recent  [E]push  [F]reduce  [G]relatively  [H]safety  [I]resistant  [J]expensive  [K]force  [L]accordingly  [M]intensity  [N]security  [O]opposed  Answers:  1.选M)。此处应填名词,作介词of的宾语,作为后置定语修饰earthquake,说明该名词是“地震”的自然属性。选项中的名词有changes,safety,intensity和security,只有intensity可以表示地震的强度,故选M)。  2.选G)。此处应填副词,用来修饰形容词less。选项中的副词有relatively“相对的”和accordingly“相应的”,第一段说这次地震和1998年的那次地震程度一样,但造成的损失却小很多,再从后句中“高速路上的车辆不是很多”可以推断:这次的地震造成的“死伤人数”与1998年比少很多,即少是相对的,故选G)relatively。  3.选A)。空格中要填的词是本句话中的主语,应为名词。这句话的主干是havestrengthenedthecity’sbuildingsandhighways,从第三段的内容和注释3的解析可知:建筑结构发生了变化,故应选择A)changes。  4.选I)。此处应填形容词,和其前面的more构成形容词的比较级,描述新型建筑与地震有关的性能。them=thecity’sbuildingsandhighways,havestrengthened"被加固"说明他们更有抵御风险的能力了。选项中的四个形容词只有resistant符合这个意思,故选I)。  5.选B)。此处应填形容词,修饰名词materials。从后面的...bentwithoutbreaking"即使弯曲也不会折断",说明是有韧性的材料。选项中的四个形容词只有flexible表示“柔韧的,可变形的”,故B)正确。  6.选F)。此处应填名词。根据上下文insertrubberandsteelbetweenthebuildinganditsfoundation“在建筑物和地基之间填充橡胶和钢材”是为了减少theimpactofgroundvibrations"地面震动对建筑物的影响"。选项中decrease和reduce都可以表示“减少”的意思,前者指稳定的逐步地减少,后者强调在大小、程度或强度方面下降或减少。故F)reduce更符合题意。  7.选D)。根据注释3,可以推知,这里要填的词是和前面的Inthepast,Laer相对应的,选项中的形容词中只有recent可表示时间,themostrecentdesighs“最近的设计”,在原文中可以对应“过去,后来”,故D)为正确答案。  8.选K)。前面提到了智能建筑,这里说明当地震使这样的建筑向前倒的时候,电脑会怎样它,使它相反方向移动。在动词push和force之间悬着。push强调的是外加的力量,而这里的拖力者是与智能建筑一体的计算机系统,故这里force符合文意。

32  9.选N)。此后应填名词,做动词offer的宾语。本句前面一直在描述新型建筑采取的抗震措施,本句是这些措施的结果。抗震型建筑为城市提供的应该是更多的安全保障,选项中表示“安全”的词有safety和security,前者是处于安全状态,不受伤害,没有危险,多用于人身,货物。后者常指国家社会的免遭战争,灾难而安然无恙,故N)security更符合文意。  10.选J)。此处应填形容词。从后面的However后的一句说明智能建筑的优点而得知,However前所说的是智能建筑的缺点。既然智能建筑很聪明,又是新事物,那么建造起来肯定会耗费很多资金的,所以选J)expensive。四级完型:  Manystudentsfindtheexperienceofattendinguniversitylecturestobeaconfusingandfrustratingexperience.Thelecturerspeaksforoneortwohours,perhaps1thetalkwithslides,writingupimportantinformationontheblackboard,2readingmaterialandgivingout3.Thenewstudentseestheotherstudentscontinuouslywritingonnotebooksand4whattowrite.Veryoftenthestudentleavesthelecture5noteswhichdonotcatchthemainpointsand6becomehardevenforthe7tounderstand.  Mostinstitutionsprovidecourseswhich8newstudentstodeveloptheskillstheyneedtobe9listenersandnote-takers.10theseareunavailable,therearemanyusefulstudy-skillsguideswhich11learnerstopracticetheseskills12.Inallcasesitisimportantto13theproblem14actuallystartingyourstudies.  Itisimportantto15thatmoststudentshavedifficultyinacquiringthelanguageskills16incollegestudy.Onewayof17thesedifficultiesistoattendthelanguageandstudy-skillsclasseswhichmostinstitutionsprovidethroughoutthe18year.Anotherbasic19istofindastudypartner20itispossibletoidentifydifficulties,exchangeideasandprovidesupport.  1.A.extendingB.illustratingC.performingD.conducting  2.A.attributingB.contributingC.distributing  D.explaining  3.A.assignmentsB.informationC.contentD.definition  4.A.suspectsB.understandsC.wondersD.convinces  5.A.withoutB.withC.onD.except  6.A.whatB.thoseC.asD.which  7.A.teachersB.classmatesC.partnersD.students  8.A.preventB.requireC.assistD.forbid  9.A.effectiveB.passiveC.relativeD.expressive  10.A.BecauseB.ThoughC.WhetherD.If

33  11.A.enableB.stimulateC.advocateD.prevent  12.A.independentlyB.repeatedly  C.logicallyD.generally  13.A.evaluateB.acquaintC.tackleD.formulate  14.A.beforeB.afterC.whileD.for 15.A.predictB.acknowledge  C.argueD.ignore  16.A.torequireB.requiredC.requiringD.arerequired  17.A.preventingB.withstanding  C.sustainingD.overcoming  18.A.averageB.ordinaryC.normalD.academic  19.A.statementB.strategyC.situationD.suggestion  20.A.inthatB.forwhichC.withwhomD.suchas   1.【答案】B  【解析】将第1,2,3题通盘考虑。此处意为“老师会花一两个小时用幻灯来解释讲课的内容,写出一些重要的信息,散发一些阅读材料,布置作业”。illustrate用图解说明,举例说明。  2.【答案】C  【解析】参考第1题答案。attribute把……的原因归为……;contribute有助于,贡献;distribute分发,散发,与下文的giveout同义。  3.【答案】A  【解析】assignments作业,任务。  4.【答案】C

34  【解析】新生发现别的学生一直在记笔记,他们不知道该记些什么,所以用wonders。而suspects(怀疑);understands(理解,明白);convinces(让人相信)与题意不符。  5.【答案】B  【解析】with结构在此表示伴随的结果,说明学生听完讲座却记了一些抓不住重点的笔记。  6.【答案】D  【解析】这里是一个并列句,并列的两个部分是whichdonotcatchthemainpointsandwhichbecomehardevenforthestudentstounderstand.  7.【答案】D  【解析】此句意为:学生记下的笔记连自己也无法明白。  8.【答案】C  【解析】鉴于上述情况,许多学校开设课程以帮助新生  培养记笔记的能力,成为一名真正有效率的听众。assist帮助,援助。  9.【答案】A  【解析】参考第8题答案。effective有效的;passive被动的;relative相对的;expressive表现的,富于表情的。  10.【答案】D  【解析】此处的意思是:如果这些课程不可行的话,还会有许多行之有效的学习技巧的指导,这些指导使学生们能够独立地锻炼这些学习技巧,在此If表示假设条件。  11.【答案】A  【解析】enablesb.todosth使人能够干什么;stimulate激发,刺激;advocate提倡,倡导;prevent阻止。  12.【答案】A  【解析】independently独立地;repeatedly再三,重复地;logically合乎逻辑地;generally大体上,一般地。  13.【答案】C

35  【解析】此句意为“通常学生在开始学习之前就应该解决这种听课技能的问题”,此处C.totackleproblem意为“解决问题”。evaluate估计,评估;acquaint使认识,了解;formulate用公式表示,系统地阐述或提出。  14.【答案】A  【解析】参考13题。  15.【答案】B  【解析】这里的意思是“承认大多数学生在获取语言技能方面有困难,这是很重要的。因为只有承认这种困难才能提出克服困难的方法”。所以选B.acknowledge承认,认可。  predict预测;argue争论,论证;ignore忽略,忽视。  16.【答案】B  【解析】过去分词做定语。  17.【答案】D  【解析】克服困难用overcomedifficulty。不用preventing阻止,阻碍;withstand经受住,抵抗;sustain支撑,经受。  18.【答案】D  【解析】本题涉及学年的表达方法。  19.【答案】B  【解析】此句意为:另一种基本的方案或策略是寻找一个学习的伙伴。strategy策略,方案。  20.【答案】C  【解析】本题测试介词与关系代词的用法,withwhom表示与同伴一起学习。四级翻译:  增词法

36  译文中添加一些原文没有的词句,表面上看似不忠实于原文,但仔细分析就会发现这些增加的词句所表达的意思并非无中生有,而是隐含在原文中的。要知道,从一种语言文字向另一种语言文字转换,有时可以找到一种语言文字在另一种语言文字中的对等词,然而要想全部依赖对等词的转换来达到翻译的目的是几乎不可能的。不同语言文字所持有的习惯决定了必须根据其中一种语言文字的习惯来适当地增词(或减词)达到多语言交际的目的。如果机械地按照字面意义直译,不仅不能表达原文的思想,精神与形象,而且还会使译文前后矛盾,闹出笑话。试想,如把“七擒孟获”机械译成tocatch/captureMengHuofor7times,不同文化背景的外国读者就很难真正理解这个典故的内涵。  例1听到你平安的消息,非常高兴!  译文:Iwasverygladonhearingthatyouwereinsafety!(增补主语)  例2勤能补拙  译文:Itisdiligencethatmakesupforthedeficiency.(增补强调句里的it)  例3对不起,打扰一下!  译文:Excusemeforinterruptingyou!(增补作宾语的代词you)  除了在翻译时要注意及时增补语法功能词之外,有时还需要增补与汉语文化内涵相关的信息。如:“三个臭皮匠,抵个诸葛亮”应当译文:ThreecobblerswiththeirwitscombinedequalZhuGeliang,themastermind.译文里themastermind就是个增补注释性表达。否则外国人不可能知道ZhuGeliang是谁。  减词法  减词是在不影响原意的情况下省略无关紧要的词语,避免拖泥带水。如“他们开始研究近况,分析近况”中的两个“近况”可以省略一个,只译成Theybegantostudyandanalyzetherecentsituation.因为原汉语句中的两个“近况”是为了强调,但英文里如果也译成两个therecentsituation,不仅显得罗嗦也不符合英文表达习惯。  例1这种制度,条件成熟的可以实行,条件不成熟的不要实行。  译文:Thissystemcanbeadoptedwhereconditionsareripeandnototherwise.(省略原文中重复出现的词语)  例2我们要解决失业问题。转自:  译文:Weshouldreduceunemployment.(省略范畴词“问题”)  例3Shelaughedandthenshruggedhershoulders.  译文:她大笑起来,然后耸了耸肩。(省略物主代词“her”)  另外,有些汉语里的词语如果译成英文反而会影响英语修饰效果,必须在译成英文时省去。如:“活到老,学到老”应该怎么翻译呢?  译文1:Ifwecouldliveanoldage,weshouldkeeplearningattheoldage.  译文2:We’dbetterkeeponlearningeventhoughweareold.  译文3:Liveandlearn.  上面三种译法,大家认为哪一种可取呢?前两种是逐字翻译,将“老”字译出来,虽然说理解上不受影响,但总觉得说话拖泥带水,松散无力。“活到老,学到老”是表示一种坚持不懈的精神,即:人无时无刻要学习。译文3没有把“老”译出来,看似不忠于原文,但却充分传达了原文鼓励大家不懈努力,坚持学习的精神,而且语言生动简练,表达有力度,也符合英语习惯。所以说,这句用减词法恰到好处。  1.Inthisway,Ibelievethatallthepeoplemaybeableto_______________(像我一样享受乘坐公车的乐趣).  2.Bypracticingthese,Ihavebeenableto_____________(在智育方面我一直能不断进步).  3.Accordingtoarecentsurvey,fourmillionpeople______________(死于与吸烟有关的疾病)eachyear.  4.______________(没有一项发明获得如此多的表扬和批评)thanInternet.  5.Anincreasingnumberofpeoplearebeginningtorealize______________(教育不能随着毕业而结束).

37  1.enjoythebusridelikeme  解析:“享受……的乐趣”直接用enjoy翻译即可。“乘坐公交车”在这里相当于一个名词短语,因此应翻译成busride而不是takethebus。如果将句子翻译成enjoytakingthebus,意思就与enjoythebusride稍有区别。前者强调动态,表示某个动作的进行,后者强调静态,表示某一经常性习惯性做法。  2.makeconstantprogressinintellectualeducation  解析:“不断进步”是指不断取得进步,“取得进步”用短语makeprogress。“不断”应选择constant。“在智育方面”作状语置于句末。www.Examw.com  3.dieofdiseaseslinkedtosmoking  解析:本句意为“由最近调查显示,每年有400万人死于与吸烟有关的疾病”。“死于……”既可以用短语dieof也可以用diefrom,但考生应注意两者用法上的区别。“与……相关”可以选用relatedto,associatedwith,linkedto。  4.Noinventionhasreceivedmorepraiseandabuse  解析:本题考点是关于“没有……比……更……”的表达,前后比较的是物——“其他发明”与“因特网”。它的基本结构是:no+sth.…morethan…。也可以用于人的比较,如:OfallthepeopleIknow,perhapsnonedeservesmyrespectmorethanMissChang,myEnglishteacher.  5.thateducationisnotcompletewithgraduation  解析:“随着……而结束/终结”要用becomplete/endwithsth.,采用的是词性转换法。将句中的动词“结束”转换成英语里的形容词complete。当然,如果大家译成动词end也是可以的,即:thateducationcannotendwithgraduation。另外,本句需填入的是一个从句,因此不能遗漏that。2012年6月英语四级模拟题四

38四级作文:  Forthispart,youareallowed30minutestowriteacompositiononthetopic:LivingaloneorLivingwithRoommates?Youshouldwriteatleast120wordsfollowingtheoutlinegivenbelowinChinese.  1.有人认为大学里应独自生活  2.另一些认为大学里应与别人同住  3.你的看法  LivingaloneorLivingwithRoommates  Nowadays,therehasappearedaheateddiscussionamongthecollegestudentsastowhethertheyshouldlivealoneoutsidethecampusorlivetogetherwithotherroommatesinthestudents’dormitory.Opinionsaredividedoverthematter.  ThosewhoareinfavorofLivingalonemaintainthatitisveryconvenienttolivebythemselves.Theycanenjoyabsolutefreedominaroomoftheirown.Theycanhavetheirowntimetablewithoutdisturbingothers.TheyarealsofreetoequiptheroomwithapersonalcomputersothattheycanhaveeasyaccesstotheInternet.  Butothersarguethatlivingwithroommateshasattractionsofitsown.Withseveralstudentssharingthesameroom,eachperson’sexperiencescanbegreatlyenriched.Theycanlearnalotfromtalkingtooneanother.Bylearningtotoleratethedifferencesbetweenindividuals,theycanbecomemoremature.  AsfarasI’mconcerned,IprefertolivewithroommatesbecauseIlovethefeelingofbelonging.Besides,itisalotcheapertoliveinadormthantorentaapartmentoutsidethecampus.  点评:本文与校园生活密切相关,也是近年来出现的现象,人们对此的看法各异,2003年后,四级作文考查加大了学校生活类题目的考查,因为这类题材不存在地域的差异,不同地域的同学都有话可说。本预测题即为校园生活类话题,与刚考完的四级作文话题类似,符合命题者出题思路,值得引起重视。四级阅读:Asthepaceoflifecontinuestoincrease,wearefastlosingtheartofrelaxation.Butrelaxationis__1__forahealthymindandbody.  Stressisanaturalpartofeverydaylifeandthereisnowayto__2__it.Infact,itisnotthebadthingasitisoftensupposedtobe.Acertainamountofstressisvitaltoprovidemotivationandgivepurposetolife.Itisonlywhenthestressgetsoutof__3__thatitcanleadtopoorperformanceandillhealth.  Theamountofstressapersoncanwithstanddependsverymuchontheindividual.Somepeoplwarenotafraidofstress,andsuch__4__areobviouslyprimematerialformanagerialresponsibilities.Othersloseheartatthefirstsightof__5__difficulties.Whenexposedtostress,inwhateverform,wereactbothphysicallyand__6__.Infactwemakechoicebetween"flightorfight"andinmore__7__daysthechoicesmadethedifferencebetweenlifeordeath.Thecriseswemeettodayareunlikelytobesoextreme,buthoweverlittlethestress,itinvolvesthesame__8__.Itiswhensuchareactionlastslong,throughcontinued__9__tostress,thathealthbecomesendangered.Sincewecannot__10__stressfromourlivesitwouldbeunwisetodosoevenifwecould,weneedtofindwaystodealwithit.  A)exposureB)charactersC)answerD)chemically

39  E)avoidF)psychologicallyG)primitiveH)transfer  I)unusualJ)controlK)removeL)escape  M)responseN)backwardO)essential  【答案】  1.选O)。此处应填形容词。前文中说人们正在失去放松的休闲方式,But转折表明作者对relaxation的重视态度,只有essential“必须的,重要的”符合文意。  2.选E)。此处应填动词。前句说Stressisanaturalpartofeverydaylife“压力是日常生活中很自然的一部分”,说明人们不能避免压力。选项中只有avoid的意思为“避免,消除”,故E)avoid正确。  3.选J)。此处应填名词。前面说有压力并不是坏事,适当的压力能给人以动力,能赋予人生活的意义。只有在什么时候,压力才会导致人们表现不佳,身体不好。根据上下文可知getoutofcontrol“压力失控”的时候,才会有不好的结果,故选J)control。  4.选B)。此处应填名词。前句中notafraidofstress“不怕压力”是一些人的性格特点,选项中可以表示人的性格特点的词只有characters,故选B)。  5.选I)。此处应填形容词,修饰名词difficulties。一些人遇到......的困难就灰心丧气,选项中的形容词中,unusual“不同寻常的”可以说明困难的程度,故选择I)。  6.选F)。此处应填副词,与physically相对应,在此physically作为“身体上的”解释。选项中的副词有chemically“化学地”和psychologically“精神地”,很明显两个副词中选择F)psychologically.就上考试大  7.选G)。此处应填形容词,修饰名词days。...days与后文中的today相对应,形成对比。选项中的是primitive,即将人类的早期和人类的今天做比较,故选G)。而N)backward说明的是社会发展的状态,不符合原文意思。  8.选M)。此处填名词。前句说,在人类的早期,选择不同就意味着生与死的差别;而现在人们碰到的危机不可能那样极端。后面but转折,说明了不管是压力的大小,人们都是在“避免”和“战斗”之间选择,下句中的suchareaction也可以说明人们对压力的反应是一样的,选项中的名词answer和response中,M)response=reaction符合文意。  9.选A)。根据上题,这种反应长时间持续的原因就是长时间的面对压力,选项中的exposure“暴露的状态,受影响”符合上下文意思,故选择A)exposure。  10.选K)。此处要填动词原形,和from构成动介搭配。人们应想办法应对压力,而不是将它......出人们的生活。选项中的transfer表示“转移”不符合原文意思;remove“移动,移除”可以和from连用,表示“除掉,移开”,符合原文意思,故选K)。  Thetypicalpre-industrialfamilynotonlyhadagoodmanychildren,butnumerousotherdependentsaswell---grandparents,uncles,auntsandcousions.Such"extended"familiesweresuitedforsurvivalinslowpaced__1__societies.Butsuchfamiliesarehardto__2__.Theyareimmobile.Industrialismdemandedmassesofworkersreadyandabletomoveoffthelandinpursuitofjobs,andtomoveagainwhenevernecessary.Thustheextendedfamily__3__sheditsexcessweightandtheso-called"nuclear"familyemerged---astripped-down,portablefamilyunit__4__onlyofparentsandasmallsetofchildren.Thisnewstylefamily,farmore__5__thanthetraditionalextendedfamily,becamethestandardmodelinalltheindustrialcounties.Super-industrialism,however,thenextstageofeco-technologicaldevelopment,__6__evenhighermobility.Thuswemayexpectmanyamongthepeopleofthefuturetocarrythestreamlinlingprocess,astePfurtherbyremainingchildren,cuttingthefamilydowntoitsmore__7__components,amanandawoman.Twopeople,perhapswithmatchedcareers,willprovemoreefficientatnavigatingthrougheducationandsocialstatus,throughjobchangesand

40geographicrelocations,thantehordinarilychild-clutteredfamily.A__8__maybethepostponementofchildren,ratherthanchildlessness.Menandwomentodayareoftentornin__9__betweenacommitmenttocareerandacommitmenttochildren.Inthefuture,many__10__willsidestePthisproblembydeferringtheentiretaskofraisingchildrenuntilafterretirement.A)transplantB)solutionC)gaduallyD)transportE)elementalF)conflictG)continuallyH)mobileI)couplesJ)agricultural)includingL)compromiseM)requiresN)primaryO)consisting  ANSWER:  1.选J)。此处应填形容词,修饰名次societies。文章第一句就说Thetypicalpre-industrialfamily...“工业化之前的典型家庭模式......”,即“extended"family存在于工业化之前的社会,即agriculturalsociety。故J)正确。选项中的primary”最初的,原始的“不符合原文意思。  2.选A)。由behardtodosth.可知,此处应填动词原形。Theyareimmobile“这样的家庭很难流动”,这说明这样的家庭arehardto...,选项中由transplant和transport两个动词原形,前者指的是“迁居,迁移”,后者指的是运输,不难推断前者符合原文意思,故A)transplant正确。3.选C)。因为这句话的句子结构完整,固此处应填副词。选项中的副词有gradually和continually,原文中...andtheso-called"nuclear"familyemerged.所谓的“核心家庭”便出现了“,emerge的意思是“浮现",强调经过一个过程后出现的。由此可以判断,这句话坏死说明核心家庭从无到有逐渐出现,而不是连续不断地出现,故选择C)gradually>  4.选O)。该句可拆分理解,即afamilyunit...ofparentsandasmellsetofchildren"一个家庭单元......父母和不多的孩子“,由此可以推出这里要填的词是表示”包含,由......组成“的。选项中的including和consisting均可以表示此意,但由原文中的of可排除including,consistof为固定刺诸,故O)为正确答案。  5.选H)。从原文中的more...than可知,此处应填形容词,构成形容词的比较级结构。由第一段最后一句可知thetraditionalextendedfamily是immobile,那么新型的家庭模式比旧的因该是更有流动性,故应选H)mobile,说明新型的家庭模式的优点。  6.选M)。此处应填动词,作这句话的谓语。Super-industrialism“(更发达的)超级工业化”......更具流动性的家庭。可推知空格处应填表示“需要,需求”的词。requires表示出自一种迫切的需要而提出的要求,很明显选择M)requires。  7.选E)。此处应填形容词,用来修饰名词components。文中说明家庭被减缩到最......的成员,即由男人和女人组成,男人和女人是一个家庭最基本的成员。选项中的E)elemental“基本的,本质的”,而N)primary“主要的,最早的”,原文强调的是男人和女人是一个家庭最basic的组成部分,故E)更符合原文意思。  8.选l)。此处应填名词。上段说两人家庭的优点,本句中提到ratherthanchildlessness“而不是不要孩子”,说明晚要孩子是解决工作和孩子的折中的办法,故选项中的L)compromise“妥协,折中”符合文意。而solution虽然也表示解决办法,但不能表达夫妇们无可奈何的心情,故排除。  9.选F)。上题中已提到工作和要孩子之间存在矛盾即conflict,根据上下文意思,这个题相对容易。  10.选I)。这段一直在说一个家庭里男人和女人因为工作和要孩子的事情发生争吵,那么将来要解决这个问题的仍然是家庭中的夫妻两人,故这里应填I)couples。  导读:Thetypicalpre-industrialfamilynotonlyhadagoodmanychildren,butnumerousotherdependentsaswell---grandparents,uncles,auntsandcousions.Such"extended"familiesweresuitedforsurvivalinslowpacedJ)agriculturalsocieties.ButsuchfamiliesarehardtoA)transplant.Theyareimmobile.(Thetypicalpre-industrialfamily="extended"family),第一段说明工业化前的大家庭模式的特点:很难流动。)

41  Industrialismdemandedmassesofworkersreadyandabletomoveoffthelandinpursuitofjobs,andtomoveagainwhenevernecessary.ThustheextendedfamilyC)graduallysheditsexcessweightandtheso-called"nuclear"familyemerged---astripped-down,portablefamilyunitO)consistingonlyofparentsandasmallsetofchildren.Thisnewstylefamily,farmoreH)mobilethanthetraditionalextendedfamily,becamethestandardmodelinalltheindustrialcounties.Super-industrialism,however,thenextstageofeco-technologicaldevelopment,M)requiresevenhighermobility.Thuswemayexpectmanyamongthepeopleofthefuturetocarrythestreamlinlingprocess,astePfurtherbyremainingchildren,cuttingthefamilydowntoitsmoreE)elementalcomponents,amanandawoman.Twopeople,perhapswithmatchedcareers,willprovemoreefficientatnavigatingthrougheducationandsocialstatus,throughjobchangesandgeographicrelocations,thantehordinarilychild-clutteredfamily.(第二段与第一段形成对比,说明industrialism“工业化”要求家庭必须流动,导致大家庭变得越来越小,甚至只剩下最基本的成员:男人和女人。)  AD)transportmaybethepostponementofchildren,ratherthanchildlessness.MenandwomentodayareoftentorninF)conflictbetweenacommitmenttocareerandacommitmenttochildren.Inthefuture,manyI)coupleswillsidestePthisproblembydeferringtheentiretaskofraisingchildrenuntilafterretirement.(最后一段说明工业化背景下,参加工作和要孩子之间存在矛盾。)  全文翻译:工业化之前的典型的家庭模式不仅包括许多孩子,而且还有很多其他的家庭成员----祖父母,叔叔,婶婶,堂兄妹等。这样的大家庭适合在慢节奏的农业社会中生存,但这样的家庭很难迁移和流动。工业化需要大量乐意并能够离开家乡外出寻找工作的工人,而且他们可以根据需要随时流动。这样,大家庭渐渐摆脱了多余的成员,而所谓的“核心家庭”便出现了:这种家庭是只包括父母和小孩的迁移方便的小家庭。这种远远比传统大家庭更易于流动的新型家庭模式便成为所有工业国家的标准模式。然而,超级工业化,即下一个阶段的生态科技的发展,要求家庭具有更大的流动性。因此我们期待:在将来,许多人采取继续进一步缩小的家庭模式。除了孩子,把家庭缩减到它的最基本成员,即只有一个男人和一个女人。夫妻两人,也许由于职业旗鼓相当,将比孩子成群的家庭在接受教育和处理社会问题方面更加爱得心应手,在更换工作类型和工作地点的方面更为有效。折中的解决办法就是推迟要孩子,而不是不要孩子。现在的男人和女人在忠于事业和抚养孩子之间总是引起冲突。在将来,许多夫妇会把全力以赴抚养孩子推迟到退休以后,以回避这一问题。  VideoconferencingisnothingmorethanatelevisionsetorPCmonitorwithacameraThroughthevideocenferencing,notonlyyourvoicebutalsoyourface,thesurroundingsandanyothergraphicandphisical__1__canbecapturedandtransmittedthroughthecommunicationsystemwithorwithoutwires.Ofcourse,whenyougointothedetails,thetechnologyinvolvedisvery__2__andthesubjectmatterlitteredwithjargon.SuchasISDN(IntegratedServicesDigitalNetwork),POTS(PlainOldTelephoneService)orthe__3__behindbandwidth,latencyandisochronywhichareusedtoexplainhowvideoconferencingworks.Goodpeoplecommunicationis__4__inanybusiness,andthemoreinteractionyoucanachieve,themorelikelyitisthatyour__5__willbetherightones.Videoconferencingnotonlyallowsyoutospeaktopeopleindifferentlocations,butalsonote__6__expressionsandgesturesthatletyouknowwhattheotherpersonisreallythinking.Meetingsaremademore__7__bysharingdocumentsandcomputerapplicationsthatasimpletelephonecannot__8__.__9__,organizationsarediscoveringthecompetitiveadvantagesandthepowerofvideoconferencing.Withadvancesinperformance,economicalpricing,theabilityto__10__essentialmeetingtoolsandconnectivitytoglobal

42telephonenetworksandstandardizedvideoconferencingprotocols,videoconferencingisnowapracticalrealityforanyorganization.  A)fortunately  B)effective  C)images  D)articulate  E)facial  F)manage  G)decisions  H)connect  I)advanced  J)integrate  K)progressive  L)concepts  M)pictures  N)increasingly  O)important  【答案】  1.选C)。空格中要填的是名词,从后面的......canbecapturedandtransmitted可以选出选项中images和pictures,pictures指“画面,图片”,一般指静态的情景;而文中的意思表达的是实时的电视会议,画面应该是动态的,images可以表示动态的形象,画面,故C)images正确。  2.选I)。这里应填形容词,修饰thetechnology。选项中的advanced,progressive,important可以用来修饰thetechnology,注释1说明了thetechnology中包含了很多行业术语,是很深奥的,排除important;而progressive“上进的,进步的”,强调寻求更好的,advanced“领先的,有高深的,并非人人都能接受的含义”,根据上下文I)advanced更符合题意。  3.选L)。首先判断这里应填名词,再根据注释2种“用于解释视频会议工作原理的......”,原文中有explain“解释,说明”可以退出,选项中的名词中concepts最符合题意。故选L)。  4.选O)。此处应填形容词。这个分句后面说交流越多,你越可能成为正确的那一个,说明交流很重要,与本土的视频会议-----一种交流的工具主题一致,这里应该填important。  5.选G)。由空格后的ones可知,这里应填名词的复数形式。前面提到themoreinteraction...,themorelikely...betherightones“得到的交互信息越多,你的......就更有可能正确的”。人们通过电视会议谈判协商,了解得信息越多,自己的判断,决策才会更正确。故这里应填G)decisions。concepts“概念”是打扰项,这里和概念无关。  6.选E)。此处应填形容词。由上文Videoconferencingnotonlyallowsyoutospeaktopeopleindifferentlocations“视频会议不仅可以让你与不同地方的人说话”,...expressionsandgesturesthatletyouknowwhattheotherpersonisreallythinking.“......表情和姿态,从而让你知道他们的真实想法”,很明显修饰expressions的应该是facial,意思是“面部表情”。  7.选B)。此处要填形容词,与more构成形容词比较级结构。从后文bysharingdocumentsandcomputerapplication“通过共享文档和计算机应用,会议能更加......”,这里电话会议的优势,也是其目的,即提高效率,故这里应填B)effective。K)progressive“进步的”并不是电话会议要达到的目的,故排除。

43  8.选F)。此处应填动词原形。选项中的动词原形有articulate,manage,integrate,connect,由原文可知,电视会议可以“共享文档和应用计算机”,而电话是不能做到的,以推断这里填的词表示“做到,实现”的词,故只有manage“设法,达成”符合文意。  9.选N)。可以推断此处应填副词,选项中有fortunately和increasingly两个副词,本文全文都在说随着技术的发达,电视电话会议越来越受欢迎,所以应该是increasingly而不是fortunately“幸运地”。  10.选J)。此处应填动词原型,宾语是各种重要的会议工具。选项中还有articulate,integrate和connect三个位动词原形。其中aiticulate作“接合”意思讲的时候指的是单独的事物由关节连接,connect“连接”表示将分离的东西连接在一起,而integrate强调的是各种事物结合,形成一个整体。integrate正确,故选J)。  Nowadays,isitpossibletotellaperson’sclassjustbylookingathim?Physicaldetails__1__tellusabouthealth,dietandtypeofworkdone.Ahundredyearsagotheworkingclassveryoftenlookdunhealthy,smallandwereeithertoothinortoofat.Theupperclasseswereoften__2__,sportingtypeswhowereusedtoagooddietandlookedhealthy.Todaylivingandworkingconditionshaveimproved,andsuch__3__wouldnolongerbesotrue.Theclothespeoplechoosetowear,however,doprovideinformationabouttheir__4__.Themostobviouswayinwhichisfortheamountofmoneyspentonthem.Expensiveclotheslookexpensiveandshowtheirwearerhadmoney.Clothescanprovideother__5__aswell.Theupperclasses__6__tobelessinterestedinfashionandweargoodqualityclothesinnon-brightcolours,madeofnaturalmateriallikewool,leatherorcotton.Lowerworkingclasspeopleoftenchooseclothesinbrightcolours,madeofman-madematerial.Asociologicalexplanationforthiswouldbethatcolorandinterestaremssingfromtheirlives,andthereforeanyopportunitytointroducethisis__7__.Clothesare__8__atapricewithinmostpeople’sreach.Newclothesmakethewearerfeelgood,andshowsome__9__ofwealthtotheoutsideworld.Todaysomenewfashionsarestartedbythelowerworkingclasspeoplewhowanttolook__10__andfeelimportant.Theywantpeopletolookatthem.来源  A)available  B)background  C)different  D)tall  E)totally  F)taken  G)descriptions  H)degree  I)clues  J)alone  K)appear  L)consider  M)full  N)hobby  O)fetched  【答案】  1.选J)。此处应填副词。可选项有totally和alone,由前一句中just可推出,外表上的细节仅仅能告诉我们这个人的健康状况,平时的营养状况以及他所从事的工作而不能看出他所处的阶级,故排除totally而选alone“惟一”。

44  2.选D)。此处应填形容词。因此此句和前一句是在将以前的工人阶级和上层阶级的身体状况进行对比,故此处应填与small“矮的”意思相反的词,选项中只有tall符合题意。而full“丰满的”不能用来修饰sportingtypes,故排除。  3.选G)。此处应填名词。现在的生活和工作条件改善了,情况已经不同了。而such指代的是上文中对一百年以前工人阶级和上层阶级的身体状况的身体状况的描述,故descriptions“描述”符合题意。  4.选B)。此处应填名词。根据句意“衣服可以提供...方面的信息”,选项中有background和hobby,选background“人的背景,社会阶层”,符合文章主题。  5.选I)。由aswell可知,此处应填名词且与本段首句中的informaiton的意思相近,选项中只有clues“线索”符合题意。  6.选K)。此处应填动词的原形。选项中有appear和consider,appear可直接接不定式;而consider带不定式的复合结构,用于主动语态时,consider后面得接宾语,故选appear。  7选F)。这句中this指代brightcolour,句子主干是anyopportunityis...。可选项有taken和fetched,take“抓住”则可以与opportunity搭配,但fetch“拿来”不能与opportunity构成动宾搭配,故排除fetched而选taken。  8.选A)。此处应填形容词。句子意思是“衣服的价钱......,大多数人都买得起。”选项中只有available“可以接受的”符合题意。  9.选H)。此处应填名词。由apricewithinmostpeople’sreach可推出,此处新衣服向外界显示的是穿衣者的富裕程度,故选择degree“程度”。  10.选C)。此处应填形容词。由newfashions和feelimportant可推出工人阶级想通过穿衣来改变他们,而选项中只有different“与众不同的”符合此意,故选择C)。  It’snevereasytoadmityouareinthewrong.Beinghuman,weallneedtoknowtheartofapologizing.Lookbackwithhonestyandthinkhowoftenyou’vejudgedroughly,said__1__things,pushedyourselfaheadattheexpenseofafriend.Thencounttheoccasionswhenyouindicatedclearlyand__2__thatyouweresorry.Abitfrightening,isn’tit?FrighteningbecausesomedeePwisdominusknowsthatwhenevenasmallwronghasbeencommitted,somemysteriousmoralfeelingis__3__,anditstaysoutofbalanceuntilfaultisacknowledgedand__4__isexpressed.Irememberadoctorfriend,tellingmeaboutamanwhocametohimwithavarietyofsigns:headaches,insomniaandstomachtrouble.No__5__causecouldbefound.Finallymyfriendsaidtotheman,"Unlessyoutellmewhat’sworryingyou,Ican’thelPyou."Aftersomehesitation,theman__6__that,asexecutorofhisfaher’swill,hehadbeencheatinghisbrother,wholivedabroad,ofhis__7__.Thenandtherethewiseolddoctormadethemanwritetohisbrotherasking__8__andenclosingachequeasthefirststePinrestoringtheirgoodrelation.Hethenwentwithhimtomailboxinthecorridor.Astheletterdisappeared,themanburstintotears."Thankyou,"Hesaid,"IthinkI’m__9__."Andhewas.Aheartfeltapologycannotonly__10__adamagedrelationshiPbutalsomakeitstronger.Ifyoucanthinkofsomeonewhodeservesanapologyfromyou,someoneyouhavewronged,orjustneglected,dosomethingaboutitrightnow.  A)heal  B)mental  C)unkind  D)regret  E)accurately  F)confessed  G)inheritance  H)physical

45  I)cured  J)treat  K)truly  L)unfaithful  M)forgiveness  N)disturbed  O)excuse  【答案】  1.选C)。此处应填形容词。从后面的pushedyourselfaheadattheexpenseofafriend可知,说的是不好的话,可选项有unkind和unfaithful,但由前面的judgedroughly“粗鲁的评判”可知这里表示说话时不考虑他人的感受,因此unkind“刻薄的”更加符合上下文语气。  2.选K)。由clearlyand一词可知,此处应填副词,与closely一起修饰道歉时的情形。选项有accurately和truly,由“回想一下你曾明确地并......地表示自己歉意的场合”,能使句意通顺的是truly“真诚地”,而accurately一般表示“(数据计算得)精确地”,故排除。  3.选N)。itstaysoutofbalanceuntil...,其中it=somemysteriousmoralfeeling可推出,出于道义上的原因,人会感到不安,故选disturbed“不安的”。而confessed“公开承认的”不符合句意。  4.选D)。此处应填名词。从“知道承认错误,表达......之后,才能觉得安心,”选项中只有regret“歉意”符合题意。  5.选H)。此处应填形容词。由headaches,insomnia(失眠)andstomachtrouble可推出,可选项有mental和physical,由朋友的话“Unless...Worryingyou,Ican’thelPyou."可知这个人是由于心理上的原因,而不是生理上的,故选physical即Nophysicalcause...。  6.选F)。此处应填动词。选项中的动词有confessed,cured和disturbed,但由Aftersomehesitation可推出,此人是在向医生坦白情况,故选择confessed“坦白”。  7.选G)。此处应填名词。由asexecutorofhisfather’swill可推知,此处说明的是,在遗产继承方面,他一直在欺骗他居住在国外的兄弟,故只有选项inheritance符合题意。  8.选M)。此处应填名词。由restoringtheirgoodrelation可推知,他是写信给他兄弟请求原谅,故选项中只有forgiveness符合题意。选项中excuse用作名词时,一般解释为“理由,借口”而没有“原谅”的意思,故排除。  9.选I)。由第二段最后一句waht’sworryingyou和这个人的感情Thankyou可推出,此处说明的是医生治好了他的心病,选项中只有cured符合题意。  10.选A)。此处应填动词的圆形。Butalsomakeitstronger“使人际关系更加稳固”,it指代adamagedrelationship,由此可知,空格处应填表示“修复”的词,选项中的heal则有“治愈,修复”的意思,符合原文意思。treat一般专指医学上的治疗,用在此处不合适。四级完型:  Mostpeoplehavenoideaofthehardworkandworrythatgosintosthecollectingofthosefascinatingbirdsandanimalsthattheypaytoseeinthezoo.Oneofthequestionsthatisalwaysaskedofmeis1Ibecameananimalcollectorinthefirst2.TheansweristhatIhavealwaysbeeninterestedinanimalsandzoos.Accordingtomyparents,thefirstwordIwasabletosaywithany3wasnottheconventional“mamma”or“daddy”,4theword“zoo”,whichIwould5overandoveragainwithashrill6untilsomeone,insgroupsto7meup,wouldtakemetothezoo.WhenI8alittleolder,welivedinGreeceandIhadagreat9ofpets,rangingfromowlstoseahorses,andIspentallmysparetime10thecountrysideinsearchoffreshspecimensto11tomycollectionofpets.12onIwentforayeartotheCityZoo,asastudent13,togetexperienceofthelargeanimals,

46suchaslions,bears,bisonandostriches,14werenoteasytokeepathome.WhenIleft,I15hadenoughmoneyofmyowntobeableto16myfirsttripandIhavebeengoing17eversincethen.Thoughacollector'sjobisnotaneasyoneandisfullof18,itiscertainlyajobwhichwillappeal19allthosewholoveanimalsand20.  1.A.howB.whereC.whenD.whether  2.A.regionB.fieldC.placeD.case  3.A.clarityB.emotionC.sentimentD.affection  4.A.exceptB.butC.exceptforD.butfor  5.A.reciteB.recognizeC.readD.repeat  6.A.volumeB.noiseC.voiceD.pitch  7.A.closeB.shutC.stopD.comfort  8.A.grewB.wasgrowingC.growD.grown  9.A.manyB.amountC.numberD.supply  10.A.livingB.cultivatingC.reclaimingD.exploring  11.A.increaseB.includeC.addD.enrich  12.A.laterB.further  C.thenD.subsequently  13.A.attendantB.keeperC.memberD.aide  14.A.whoB.theyC.ofwhichD.which  15.A.luckilyB.gladlyC.nearlyD.successfully  16.A.payB.provideC.allowD.finance  17.A.normallyB.regularlyC.usuallyD.often  18.A.expectationsB.sorrows  C.excitementD.disappointments

47  19.A.forB.withC.toD.from  20.A.excursionB.travelC.journeyD.Trip  1.【答案】A  【解析】根据下一句及随后的内容,作者讲的是怎样成为动物爱好者的(从小就喜欢动物),应当选择A.how。  2.【答案】C  【解析】inthefirstplace是固定短语,意思是“首先”。此句意思是:别人经常问到的问题之一,是当初我是如何爱上动物的。  3.【答案】A  【解析】这句话的意思是:作者在呀呀学语之时,最早发清楚的音是“zoo”(动物园),而不是“妈妈”,“爸爸”,因此,应选clarity“清晰”。填入其他选项emotion(感情),sentiment(多愁善感),affection(友爱)不合逻辑。  4.【答案】B  【解析】but在此处连接另一个句子(itwastheword“zoo”,“itwas”被省略),表示转折,意为“而是”,Except,exceptfor,butfor的用法接近,表示“除了……”。  例如:  WegothereeverydayexceptSunday.  Heansweredallthequestionsexceptforthelastone.  Forayearthedamremainedcompletebutforthegenerators.  从意思上,逻辑上,都应当选but。  5.【答案】D  【解析】根据后面的overandoveragain,应选“repeat”  6.【答案】C  【解析】小孩想去动物园,便不停地发出尖叫声,故选“voice”。Ashrillvoice与scream的意思接近。volume(音量);noise(噪音);pitch(音调)均不合要求。

48  7.【答案】B  【解析】shutsb.up是指让某人住口,为了让孩子停止尖叫,只好带他去动物园。  8.【答案】A  【解析】根据后面主句的时态,此处只能用一般过去时。  9.【答案】C  【解析】agreatmany后直接跟可数名词的复数形式;agreat/largeamountof后跟不可数名词;只有agreat/largenumberof后可以跟可数名词的复数形式。  例如:  IreadagreatmanyEnglishbooks.  Alargeamountofmoneyisspentontobaccoeveryyear.  Agreatnumberofciviliansweremurderedincoldblood.  10.【答案】D  【解析】living后必须接介词in,意为“居住”;cultivating耕种;reclaiming开垦;只有exploring有探察的意思。  11.【答案】C  【解析】addto相当于increase,增加。其余选项后面都不接to。  12.【答案】A  【解析】lateron为固定短语,“后来”。  13.【答案】D  【解析】attendant仆人;keeper可理解为“饲养员”,但是astudentkeeper容易被误解为“收留学生的人”;aide有“助手”之义。作者一边上学,一边在动物园里打工,只能当助手。  14.【答案】D  【解析】which在此引导定语从句,修饰前面列出的动物。  15.【答案】D

49  【解析】因为钱是在动物园打工挣的,选successfully更能体现其含义。  16.【答案】D  【解析】financemyfirsttrip意为“支付我的旅行  费用”;pay后应接介词for;其他选项的意思相差甚远。  17.【答案】B  【解析】此句为现在完成进行时态,选regularly比较贴切。  18.【答案】D  【解析】此句是由though引导的让步状语从句,应与主句意思相对立。选项中,sorrows和disappointments与主句的appealto相对立,但sorrows的分量太重。  19.【答案】C  【解析】appealto为成语,意思是“吸引”。  20.【答案】B  【解析】excursion短途旅行;journey(从一地到另一地的)长距离,具体的旅途;travel旅行,旅游,海外旅行;trip旅行,远足。四级翻译:1._____________(没有什么可做),weplayedgames.2.Henotonly__________(把枯燥的工作强加给我),butalsotookawayallourtips.3.___________(在这种情况下),Icouldn'tgoaway.4.ThepopulationofAfricaisgrowing_________(极其迅速)5.Thegirlistooyoungto_____________(和她的父母分开).1.答案:Therebeingnothingtodo[解析]独立主格结构:there+being+其他成分,多放在句首。2.答案:imposeddullworkonme[解析]1)词组:impose…onsb.(把……强加给某人)2)时态:根据后半句,前半句也应使用一般过去时。3.答案:Under/Inthe/thesecircumstances[解析]词组:under/inthe/thesecircumstances(在这种情况下)4.答案:by/inleapsandbounds[解析]词组:by/inleapsandbounds(极其迅速)5.答案:beseparatedfromherparents

50[解析]1)语态:主语是thegirl,应使用被动语态。2)词组:separatesb./sth.from/使某人/某物与……分离)2012年6月英语四级模拟题五

51四级作文:  Directions:Forthispart,youareallowed30minutestowriteashortessayonthetopic  Advertisement.Youshouldwriteatleast120wordsfollowingtheoutlinegivenbelowinChinese:  1.广告的作用  2.广告的形式多样  3.广告的夸张性  Advertisement  Advertisementsareforcingtheirwayintopeople’slives.Peoplerefertoadvertisementsintheirdailylivesbecausetheyareconsumers.Theadvertisersareusuallymanufacturers,retailersandsalesmen.Theirmerchandiseneedstobeadvertisedtobringittotheattentiontothecustomers.Thusnearlyeveryproductisadvertisedinsomeway.Toalargeextent,goodadvertisingleadstosuccesswhilebadadvertisingcanmeanfailure.  Therearemanywaystoadvertiseand‘ads’comeindifferentforms.Newspaperscarryadvertisements.SomeproductsarepublicizedonTVandradiowhichbringthemintonoticeofawideaudience.Billboardsalsocarryadvertising.Advertisingisabigindustrynowandmanyagencieshavebeensetuptofurnishavarietyofforms..  However,advertisingisnotalwaystruthful.Aproductisoftenmisrepresented.Theadvertiserexaggeratesthebenefitsofthemerchandisehewantstosell.Thus,hemisrepresentsthetruth.Theconsumerfallsvictimtosuchadvertising.Millionsofpeoplehaveboughtadvertisedproductsandhavebeendissatisfiedwiththem。  点评:这是一篇说明文,用说明的表达方式来解说事物,阐明事理。写好说明文,不仅要抓住特征,注意条理,而且要巧妙运用说明方法,像下定义、举例子、作比较、分类别、列数字、打比方等四级阅读:BaekelandandHartmannreportthatthe“shortsleepers”hadbeenmoreorlessaverageintheirsleepneedsuntilthemenwereintheirteens.Butataboutage15orso,themenvoluntarilybegancuttingdowntheirnightlysleeptimebecauseofpressuresfromschool,work,andotheractivities.Thesementendedtoviewtheirnightlyperiodsofunconsciousnessasbothersomeinterruptionsintheirdailyroutines.  Ingeneral,these“shortsleeps”appearedambitious,active,energetic,cheerful,conformist(不动摇)intheiropinions,andverysureabouttheircareerchoices.Theyoftenheldseveraljobsatonce,orworkersfull-orpart-timewhilegoingtoschool.Andmanyofthemhadastrongurgetoappear“normal”or“acceptable”totheirfriendsandassociates.  Whenaskedtorecalltheirdreams,the“shortsleepers”didpoorly.Morethanthis,theyseemedtoprefernotremembering.Insimilarfashion,theirusualwayofdealingwithpsychologicalproblemswastodenythattheproblemexisted,andthentokeepbusyinthehopethatthetroublewouldgoaway.  Thesleeppatternsofthe“shortsleepers”weresimilarto,butlessextremethan,sleeppatternsshownbymanymentalpatientscategorizedasmanic(疯人).  The“longsleepers”werequitedifferentindeed.BaekelandandHartmannreportthattheseyoungmenhadbeenlengthysleepssincechildhood.Theyseemedtoenjoytheirsleep,protectedit,andwerequiteconcernedwhentheywereoccasionallydeprivedoftheirdesired9hoursofnightly

52bedrest.Theytendedtorecalltheirdreamsmuchbetterthandidthe“shortsleepers.”  Manyofthe“longsleepers”wereshy,anxious,introverted(内向),inhibited(压抑),passive,mildlydepressed,andunsureofthemselves(particularlyinsocialsituations).Severalopenlystatesthatsleepwasanescapefromtheirdailyproblems.  1.Accordingtothereport,______.  A)manyshortsleepersneedlesssleepbynature  B)manyshortsleepersareobligedtoreducetheirnightlysleeptimebecausetheyarebusywiththeirwork  C)longsleeperssleepalongerperiodoftimeduringtheday  D)manylongsleeperspreservetheirsleepinghabitformedduringtheirchildhood  2.Many“shortsleepers”arelikelytoholdtheviewthat_____.  A)sleepisawithdrawalfromthereality  B)sleepinterfereswiththeirsoundjudgement  C)sleepistheleastexpensiveitemontheirroutineprogram  D)sleepisthebestwaytodealwithpsychologicaltroubles  3.Itisstatedinthethirdparagraphthatshortsleepers_____.  A)areideallyvigorousevenunderthepressuresoflife  B)oftenneglecttheconsequencesofinadequatesleep  C)donotknowhowtorelaxproperly  D)aremoreunlikelytorunintomentalproblems  4.Whensometimestheycannotenjoyadequatesleep,thelongsleepersmight____.  A)appeardisturbed  B)becomeenergetic  C)feeldissatisfied  D)beextremelydepressed  5.WhichofthefollowingisNotincludedinthepassage?  A)Ifonesleepsinadequately,hisperformancesuffersandhismemoryisweakened  B)Thesleeppatternsofshortsleepersareexactlythesaneasthoseshownbymanymentalpatients  C)Longandshortsleepersdifferintheirattitudestowardssleep  D)Shortsleeperswouldbebetteroffwithmorerest  【答案及详解】  答案:DCBAB  贝克尔和哈特曼报道说,“睡眠少的人”在未进入少年期之前,其正常睡眠时间大致与所需要的时间差不多。但到了15岁左右,由于学校、工作或其它活动的地压力,他们就故意地减少了夜间睡眠的时间。这些人持有这样的观点:夜间睡眠是一件令人讨厌的事情,打断了日常事务。  总的说来,这些“睡眠少的人”表现得雄心勃勃、积极活跃、精力充沛、无意识乐观豁达、立场坚定,对自己职业的选择胸有成竹。他们往往同时从事几项工作,或者一边上学读书,一边从事专职或兼职工作。其中许多人有强烈愿望,想在朋友和熟人面前表现得“正常”或“合群”。  当让他们回忆梦境时,“睡眠少的人”回忆不起什么来。更有甚者,他们似乎情愿什么都记不住。类似的情况是他们通常处理心理问题的方式:不承认问题的存在,希望只要忙忙碌碌,麻烦总会过去的。

53  “睡眠少的人”的睡眠模式与被划入疯子之类精神病患者的睡眠模式十分相似,只不过没有那么严重而已。  “睡眠多的人”情形则大不相同。贝克尔和哈特曼报道说,这些年轻人从小的,有抱负的睡眠就一直很长。他们好像注重睡眠,不让睡眠受打搅。偶尔没有所需的9个小时夜间卧床休息,他们便会十分不安。他们比“睡眠少的人”要更能回忆得起梦的内容。许多“睡眠多的人”腼腆、焦躁、内向、压抑、消极和稍微有点儿沮丧,尤其在社交场合缺乏自信。好几个人坦言,睡眠是摆脱每天烦恼的一种方式。  Nowletuslookathowweread.Whenwereadaprintedtext,oureyesmoveacrossapageinshort,jerkymovement.Werecognizewordsusuallywhenoureyesarestillwhentheyfixate.Eachtimetheyfixate,weseeagroupofwords.Thisisknownastherecognitionspanorthevisualspan.Thelengthoftimeofrwhichtheeyesstop---thedurationofthefixation----variesconsiderablyfrompersontoperson.Italsovaieswithinanyonepersonaccordingtohispurposeinreadingandhisfamiliaritywiththetext.Furthermore,itcanbeaffectedbysuchfactorsaslightingandtiredness.  Unfortunately,inthepast,manyreadingimprovementcourseshaveconcentratedtoomuchonhowoureyesmoveacrosstheprintedpage.Asaresultofthismisleadingemphasisonthepurelyvisualaspectsofreading,numerousexerciseshavebeendevisedtotraintheeyestoseemorewordsatonefixation.Forinstance,insomeexercises,wordsareflashedontoascreenfor,say,atenthoratwentiethofasecond.Oneoftheexerciseshasrequiredstudentstofixtheireyesonsomecentralpoint,takinginthewordsoneitherside.Suchwordpatternsareoftenconstructedintheshapeofrathersteeppyramidssothereadertakesinmoreandmorewordsateachsuccessivefixation.Alltheseexercisesareveryclever,butit’sonethingtoimproveaperson’sabilitytoseewordsandquiteanotherthingtoimprovehisabilitytoreadatextefficiently.Readingrequirestheabilitytounderstandtherelationshipbetweenwords.Consequently,forthesereasons,manyexpertshavenowbeguntoquestiontheusefulnessofeyetraining,especiallysinceanyapproachwhichtrainsapersontoreadisolatedwordsandphraseswouldseemunlikelytohelphiminreadingacontinuoustext.  Q:  1.Thetimeoftherecognitionspancanbeaffectedbythefollowingfactsexcept________.  A.one’sfamiliaritywiththetext  B.one’spurposeinreading  C.thelengthofagroupofwords  D.lightingandtiredness  2.Theauthormaybelievethatreading______.  A.requiresareadertotakeinmorewordsateachfixation  B.requiresareadertoseewordsmorequickly  C.demandsandeeply-participatingmind  D.demandsmoremindthaneyes  3.Whatdoestheauthormeanbysaying“butit’sonethingtoimproveaperson’sabilitytoseewordsandquiteanotherthingtoimprovehisabilitytoreadatextefficiently.”inthesecondparapraph?  A.Theabilitytoseewordsisnotneededwhenanefficientreadingisconducted.  B.Thereadingexercisesmentionedcan’thelptoimproveboththeabilitytoseeandtocomprehendwords.

54  C.Thereadingexercisesmentionedcan’thelptoimproveanefficientreading.  D.Thereadingexercisesmentionedhasdoneagreatjobtoimproveone’sabilitytoseewords.  4.WhichofthefollowingisNOTtrue?  A.Thevisualspanisawordoragroupofwordsweseeeachtime.  B.Manyexpertsbegantoquestiontheefficiencyofeyetraining.  C.Theemphasisonthepurelyvisualaspectsismisleading.  D.Theeyetrainingwillhelpreadersinreadingacontinuoustext.  5.Thetuneoftheauthorinwritingthisarticleis________  Acritical  Bneutral  Cprssimistic  Doptimistic  【答案及详解】  答案:CCCDA  1C。事实细节题。第一段提到了影响视幅的因素:不同的人,不同的阅读目的,对材料的熟悉程度,光线,疲劳。C“一组词的长度”不是能影响视幅的因素,是本题的答案。  2C。观点态度题。作者在第二段第二句提到,眼睛训练课程只注重了阅读的视觉因素。倒数第二句书哦,阅读要求具备理解单词间关系的能力。因此选项C“阅读需要大脑的深度参与”正确。作者没有否定阅读的视觉因素的必要性,但是也没有说读者应该练习拓宽视幅,加快阅读速度。所以AB两项都不正确。作者没有对大脑和眼睛在阅读过程中的重要性进行对比,所以不选D  3C。作者在这句话中先是肯定了那些阅读练习提高看单词的能力,然后提出了有效阅读概念。后面句中还指出,有效阅读需要的是理解单词间的联系的能力。所以作者的意思应该是那些(训练眼睛的)阅读练习对与有效的阅读无益。B与文章相反。D句是蕴涵其中一个意思,却不是作者想表达的方向。  4D。第二段最后一句说,眼睛训练对于帮助读者阅读连贯文章无益,所以D是错的,是本题答案。  5A。参考前面的结构剖析,作者写本文的主要目的是对那些只关注阅读的视觉因素的阅读能力课程进行批判,所以答案应该是A。  Asisknowntoall,theorganizationandmanagementofwagesandsalariesareverycomplex.Generallyspeaking,theAccountsDepartmentis__1__forcalculationsofpay,whilethePersonnelDepartmentisinterestedindiscussionswiththeemployeesaboutpay.  Ifafirmwantsto__2__anewwageandsalarystructure,itisessentialthatthefirmshoulddecideona__3__ofjobevaluationandwaysofmeasuringtheperformanceofitsemployees.Inordertobe__4__,thatnewpaystructurewillneedagreementbetweenTradeUnionsandemployers.Injobevaluation,alloftherequirementsofeachjobaredefinedinadetailedjobdescription.Eachofthsoerequirementsisgivenavalue,usuallyin"points",whichare__5__togethertogiveatotalvalueforthejob.Formiddleandhighermanagement,aspecialmethodisusedtoevaluatemanagersontheirknowledgeofthejob,theirresponsibility,andtheir__6__tosolveproblems.Becauseofthedifficultyinmeasuringmanagementwork,however,jobgradesformanagersareoftendecidedwithout__7__toanevaluationsystembasedonpoints.  Inattemptingtodesignapaysystem,thePersonnelDepartmentshould__8__thevalueofeachjobwiththeseinthejobmarket.__9__,paymentforajobshouldvarywithanydifferences

55inthewaythatthejobisperformed.Whereitissimpletomeasuretheworkdone,asintheworksdonewithhands,monetaryencouragementschemesareoftenchosen,for__10__workers,wheremeasurementisdifficult,methodsofadditionalpaymentsareemployed.  [A]compare  [B]responsible  [C]useful  [D]added  [E]find  [F]reference  [G]indirect  [H]method  [I]successful  [J]combined  [K]Necessarily  [L]capacity  [M]ability  [N]Basically  [O]adopt  【答案及详解】  1.选B)。此处应填形容词。原文意思为“会计部门...计算报酬”,选项中的形容词responsible“负责任的”,useful“有用的”,indirect“间接的”,successful“成功的”,其中indirect不能与for连用,排除;剩余几项中只有B)responsible意义符合原句,其他均不符合,故排除。  2.选O)。此处应填动词原形。本文主题就是采用一种新的工资和薪水制度时需要注意的问题,选项中的动词原形有compare“比较”,find“找到”,adopt“采纳”,分别带入原文,只有adopt最符合原文意思,故选O).  3.选H)。此处应填名词。从原文看,and连接并列结构,所以要填的词应与and后面的ways意思一致,选项中的名词只有mathod=ways,故选H)method。  4.选I)。此处应填形容词。上文说新的工资制度需要一套决定工作评估和衡量雇员表现的方法,说的是制度“是否有用”的问题。这句说的时新的工资制度执行过程中的问题,劳资双方先期达成一致是薪酬制度成功的必要条件。形容词useful和successful,I)seccessful更符合原文意思。  5.选D)。此处应填动词。这个动词的宾语是point“分值”,把分值...起来togiveatotalvalue"得出总分值",选项中有added和combined,前者指“把...相加”,后者意为“把...结合在一起”,原文指将分值相加得出总分,故D)added最符合文意。  6.选M)。此处应填名词。首先solveproblem“解决问题”是经理们应具备的能力,选项中的capacity与ability都可以表示“能力”;前者强调的是理解的能力和接受事物的能力,而后者强调实际应用的能力,故不难判断解决问题的能力应该用M)ability。  7.选F)。此处应填名词。前半句指出“因为管理工作很难评估,经理的工作得分不需要参照基于分值的评估系统决定。”withoutreferenceto为固定搭配,意思是“与...无关”。故选F)reference。考试大-中国教育考试门户网站(www.233.com)  8.选A)。此处应填动词原形。原句中出现了thevalueofeachjob“每种工作的价值”和theseinthejobmarket“工作市场上的(工作价值)”,说明人事部门通过比较两种价值来计算工资制度。选项中只有compare表示“比较”,所以A)正确。

56  9.选K)。此处应填副词,修饰整句话。“报酬要随工作表现各方面的不同而有所改变”,选项中Necessarily“必须地”与Basically“基本地”为副词,分别带入原文,“报酬...的变化是必须的”更符合上下文意思,故排除N),选K)。  10.选G)。此处应填形容词。前文中提到一种情况即simpletomeasuretheworkdone"易于评估的工作",通常用现金奖励办法;而...measurementisdifficult“难于评估的”,说明这些工作人员的工作不像手工工作那么直接,因而是间接的,故选项中只有G)indirect符合原句的意思。  Nowletuslookathowweread.Whenwereadaprintedtext,oureyesmoveacrossapageinshort,jerkymovement.Werecognizewordsusuallywhenoureyesarestillwhentheyfixate.Eachtimetheyfixate,weseeagroupofwords.Thisisknownastherecognitionspanorthevisualspan.Thelengthoftimeofrwhichtheeyesstop---thedurationofthefixation----variesconsiderablyfrompersontoperson.Italsovaieswithinanyonepersonaccordingtohispurposeinreadingandhisfamiliaritywiththetext.Furthermore,itcanbeaffectedbysuchfactorsaslightingandtiredness.  Unfortunately,inthepast,manyreadingimprovementcourseshaveconcentratedtoomuchonhowoureyesmoveacrosstheprintedpage.Asaresultofthismisleadingemphasisonthepurelyvisualaspectsofreading,numerousexerciseshavebeendevisedtotraintheeyestoseemorewordsatonefixation.Forinstance,insomeexercises,wordsareflashedontoascreenfor,say,atenthoratwentiethofasecond.Oneoftheexerciseshasrequiredstudentstofixtheireyesonsomecentralpoint,takinginthewordsoneitherside.Suchwordpatternsareoftenconstructedintheshapeofrathersteeppyramidssothereadertakesinmoreandmorewordsateachsuccessivefixation.Alltheseexercisesareveryclever,butit’sonethingtoimproveaperson’sabilitytoseewordsandquiteanotherthingtoimprovehisabilitytoreadatextefficiently.Readingrequirestheabilitytounderstandtherelationshipbetweenwords.Consequently,forthesereasons,manyexpertshavenowbeguntoquestiontheusefulnessofeyetraining,especiallysinceanyapproachwhichtrainsapersontoreadisolatedwordsandphraseswouldseemunlikelytohelphiminreadingacontinuoustext.  Q:  1.Thetimeoftherecognitionspancanbeaffectedbythefollowingfactsexcept________.  A.one’sfamiliaritywiththetext  B.one’spurposeinreading考试大-中国教育考试门户网站(www.233.com)  C.thelengthofagroupofwords  D.lightingandtiredness  2.Theauthormaybelievethatreading______.  A.requiresareadertotakeinmorewordsateachfixation  B.requiresareadertoseewordsmorequickly  C.demandsandeeply-participatingmind  D.demandsmoremindthaneyes  3.Whatdoestheauthormeanbysaying“butit’sonethingtoimproveaperson’sabilitytoseewordsandquiteanotherthingtoimprovehisabilitytoreadatextefficiently.”inthesecondparapraph?  A.Theabilitytoseewordsisnotneededwhenanefficientreadingisconducted.  B.Thereadingexercisesmentionedcan’thelptoimproveboththeabilitytoseeandtocomprehendwords.  C.Thereadingexercisesmentionedcan’thelptoimproveanefficientreading.

57  D.Thereadingexercisesmentionedhasdoneagreatjobtoimproveone’sabilitytoseewords.  4.WhichofthefollowingisNOTtrue?  A.Thevisualspanisawordoragroupofwordsweseeeachtime.  B.Manyexpertsbegantoquestiontheefficiencyofeyetraining.  C.Theemphasisonthepurelyvisualaspectsismisleading.  D.Theeyetrainingwillhelpreadersinreadingacontinuoustext.  5.Thetuneoftheauthorinwritingthisarticleis________  A.critical  B.neutral  C.prssimistic  D.optimistic  【答案及详解】  答案:CCCDA  解题思路  1C。事实细节题。第一段提到了影响视幅的因素:不同的人,不同的阅读目的,对材料的熟悉程度,光线,疲劳。C“一组词的长度”不是能影响视幅的因素,是本题的答案。  2C。观点态度题。作者在第二段第二句提到,眼睛训练课程只注重了阅读的视觉因素。倒数第二句书哦,阅读要求具备理解单词间关系的能力。因此选项C“阅读需要大脑的深度参与”正确。作者没有否定阅读的视觉因素的必要性,但是也没有说读者应该练习拓宽视幅,加快阅读速度。所以AB两项都不正确。作者没有对大脑和眼睛在阅读过程中的重要性进行对比,所以不选D  3C。作者在这句话中先是肯定了那些阅读练习提高看单词的能力,然后提出了有效阅读概念。后面句中还指出,有效阅读需要的是理解单词间的联系的能力。所以作者的意思应该是那些(训练眼睛的)阅读练习对与有效的阅读无益。B与文章相反。D句是蕴涵其中一个意思,却不是作者想表达的方向。  4D。第二段最后一句说,眼睛训练对于帮助读者阅读连贯文章无益,所以D是错的,是本题答案。  5A。参考前面的结构剖析,作者写本文的主要目的是对那些只关注阅读的视觉因素的阅读能力课程进行批判,所以答案应该是A。  VideoconferencingisnothingmorethanatelevisionsetorPCmonitorwithacameraThroughthevideocenferencing,notonlyyourvoicebutalsoyourface,thesurroundingsandanyothergraphicandphisical__1__canbecapturedandtransmittedthroughthecommunicationsystemwithorwithoutwires.Ofcourse,whenyougointothedetails,thetechnologyinvolvedisvery__2__andthesubjectmatterlitteredwithjargon.SuchasISDN(IntegratedServicesDigitalNetwork),POTS(PlainOldTelephoneService)orthe__3__behindbandwidth,latencyandisochronywhichareusedtoexplainhowvideoconferencingworks.Goodpeoplecommunicationis__4__inanybusiness,andthemoreinteractionyoucanachieve,themorelikelyitisthatyour__5__willbetherightones.Videoconferencingnotonlyallowsyoutospeaktopeopleindifferentlocations,butalsonote__6__expressionsandgesturesthatletyouknowwhattheotherpersonisreallythinking.Meetingsaremademore__7__bysharingdocumentsandcomputerapplicationsthatasimpletelephonecannot__8__.__9__,organizationsarediscoveringthecompetitiveadvantagesandthepowerofvideoconferencing.Withadvancesinperformance,economicalpricing,theabilityto__10__essentialmeetingtoolsandconnectivitytoglobal

58telephonenetworksandstandardizedvideoconferencingprotocols,videoconferencingisnowapracticalrealityforanyorganization.  A)fortunately  B)effective  C)images  D)articulate  E)facial  F)manage  G)decisions  H)connect  I)advanced  J)integrate  K)progressive  L)concepts  M)pictures  N)increasingly  O)important  【答案】  1.选C)。空格中要填的是名词,从后面的......canbecapturedandtransmitted可以选出选项中images和pictures,pictures指“画面,图片”,一般指静态的情景;而文中的意思表达的是实时的电视会议,画面应该是动态的,images可以表示动态的形象,画面,故C)images正确。  2.选I)。这里应填形容词,修饰thetechnology。选项中的advanced,progressive,important可以用来修饰thetechnology,注释1说明了thetechnology中包含了很多行业术语,是很深奥的,排除important;而progressive“上进的,进步的”,强调寻求更好的,advanced“领先的,有高深的,并非人人都能接受的含义”,根据上下文I)advanced更符合题意。  3.选L)。首先判断这里应填名词,再根据注释2种“用于解释视频会议工作原理的......”,原文中有explain“解释,说明”可以退出,选项中的名词中concepts最符合题意。故选L)。  4.选O)。此处应填形容词。这个分句后面说交流越多,你越可能成为正确的那一个,说明交流很重要,与本土的视频会议-----一种交流的工具主题一致,这里应该填important。  5.选G)。由空格后的ones可知,这里应填名词的复数形式。前面提到themoreinteraction...,themorelikely...betherightones“得到的交互信息越多,你的......就更有可能正确的”。人们通过电视会议谈判协商,了解得信息越多,自己的判断,决策才会更正确。故这里应填G)decisions。concepts“概念”是打扰项,这里和概念无关。  6.选E)。此处应填形容词。由上文Videoconferencingnotonlyallowsyoutospeaktopeopleindifferentlocations“视频会议不仅可以让你与不同地方的人说话”,...expressionsandgesturesthatletyouknowwhattheotherpersonisreallythinking.“......表情和姿态,从而让你知道他们的真实想法”,很明显修饰expressions的应该是facial,意思是“面部表情”。  7.选B)。此处要填形容词,与more构成形容词比较级结构。从后文bysharingdocumentsandcomputerapplication“通过共享文档和计算机应用,会议能更加......”,这里电话会议的优势,也是其目的,即提高效率,故这里应填B)effective。K)progressive“进步的”并不是电话会议要达到的目的,故排除。

59  8.选F)。此处应填动词原形。选项中的动词原形有articulate,manage,integrate,connect,由原文可知,电视会议可以“共享文档和应用计算机”,而电话是不能做到的,以推断这里填的词表示“做到,实现”的词,故只有manage“设法,达成”符合文意。  9.选N)。可以推断此处应填副词,选项中有fortunately和increasingly两个副词,本文全文都在说随着技术的发达,电视电话会议越来越受欢迎,所以应该是increasingly而不是fortunately“幸运地”。  10.选J)。此处应填动词原型,宾语是各种重要的会议工具。选项中还有articulate,integrate和connect三个位动词原形。其中aiticulate作“接合”意思讲的时候指的是单独的事物由关节连接,connect“连接”表示将分离的东西连接在一起,而integrate强调的是各种事物结合,形成一个整体。integrate正确,故选J)。四级完型:  WhowontheWorldCup1994footballgame?WhathappenedattheUnitedNations?Howdidthecriticslikethenewplay?1aneventtakesplace,newspapersareonthestreets2thedetails.Whereveranythinghappensintheworld,reportsareonthespotto3thenews.  Newspapershaveonebasic4,togetthenewsasquicklyaspossiblefromitssource,fromthosewhomakeittothosewhowantto5it.Radio,telegraph,television,and6inventionsbroughtcompetitionfornewspapers.Sodidthedevelopmentofmagazinesandothermeansofcommunication.7,thiscompetitionmerelyspurredthenewspaperson.Theyquicklymadeuseofthenewerandfastermeansofcommunicationtoimprovethe8andthustheefficiencyoftheirownoperations.Todaymorenewspapersare9andreadthaneverbefore.Competitionalsolednewspaperstobranchoutsintosmanyotherfields.Besideskeepingreaders10ofthelatestnews,today'snewspapers11andinfluencereadersaboutpoliticsandotherimportantandseriousmatters.Newspapersinfluencereaders'economicchoices12advertising.Mostnewspapersdependonadvertisingfortheirvery13.Newspapersaresoldatapricethat14evenasmallfractionofthecostofproduction.Themain15ofincomeformostnewspapersiscommercialadvertising.The16insellingadvertisingdependsonanewspaper'svaluetoadvertisers.This17intermsofcirculation.Howmanypeoplereadthenewspaper?Circulationdepends18ontheworkofthecirculationdepartmentandontheservicesorentertainment19inanewspaper'spages.Butforthemostpart,circulationdependsonanewspaper'svaluetoreadersasasourceofinformation20thecommunity,city,country,state,nation,andworld—andevenouterspace.  1.A.JustwhenB.WhileC.SoonafterD.Before  2.A.togiveB.givingC.givenD.beinggiven  3.A.gatherB.spreadC.carryD.bring  4.A.reasonB.causeC.problemD.purpose  5.A.makeB.publishC.knowD.write  6.A.anotherB.otherC.oneanotherD.theother  7.A.HoweverB.AndC.ThereforeD.So

60  8.A.valueB.ratioC.rateD.speed  9.A.spreadB.passedC.printedD.completed  10.A.informB.beinformed  C.tobeinformedD.informed  11.A.entertainB.encourageC.educateD.edit  12.A.onB.throughC.withD.of  13.A.formsB.existenceC.contentsD.purpose  14.A.triestocoverB.managestocover  C.failstocoverD.succeedsin  15.A.sourceB.originC.courseD.finance  16.A.wayB.meansC.chanceD.success  17.A.measuresB.measured  C.IsmeasuredD.wasmeasured  18.A.somewhatB.littleC.muchD.something  19.A.offeringB.offered  C.whichofferedD.tobeoffered  20.A.byB.withC.atD.about  ClozeTest3  1.【答案】A  【解析】just在此为副词,意为“刚刚”,做状语。此句意为“一个事件刚刚发生,街上就有报纸报道详情了。说明报纸对新闻的反应之快。  2.【答案】A  【解析】to

61give和giving都合乎语法,但giving强调的是正在发生的动作,而此处重点表达的是“反应快”,不是正在做什么。  3.【答案】A  【解析】消息,信息要靠收集。  4.【答案】D  【解析】后面的不定式短语表示目的。  5.【答案】C  【解析】提供信息的目的是为了让他人知道,所以选C。  6.【答案】B  【解析】other意为“其他的”。此句意为:无线电,电报,电视,及其它发明,成为报纸的竞争对手。  7.【答案】A  【解析】根据句中的merely及其后所述内容,应选however,表转折。  8.【答案】D  【解析】使用更新,更快的通讯工具,目的是提高速度。  9.【答案】C  【解析】报纸是印出来的,先印后看(读)。  10.【答案】D  【解析】keepsb.+过去分词是一种复合结构,sb.与过去分词为被动关系,意为保持这种关系的继续。此句的意思是:报纸不断地为读者提供新闻信息。  11.【答案】C  【解析】关于politics之类的严肃话题,只能选educate。  12.【答案】B  【解析】此句意为:报纸通过广告影响读者在经济生活中的选择。

62  13.【答案】B  【解析】大多数报纸依靠广告收入来维持生存,此现象人人皆知。  14.【答案】C  【解析】报纸的售价之低,不足以抵付成本的一小部分。符合上下文关于广告收入的说法。  15.【答案】A  【解析】收入来源应该用source。因为source指河流,泉水的发源地;常指抽象事物的根源或来源以及资料,信息的出处或来源。origin起源,起因。指事物后来发生,发展变化的最初起点,或指人的出身和血统。  16.【答案】D  【解析】succeedin为固定短语。此句意为:广告业务的成功,取决于报纸在客户(要打广告的人)心中的价值。  17.【答案】C  【解析】根据上下文,此处应该用一般现在时的被动语态,此句意为:报纸在客户心中的价值,是靠发行量衡量的。  18.【答案】C  【解析】该句意为:发行量的大小,很大程度上取决于发行部门的工作及报纸所提供的服务功能和娱乐功能。  19.【答案】B  【解析】offered作services和entertainment的定语。  20.【答案】D  【解析】information后面接介词about,表示“关于”。四级翻译:强化训练三1.____________(位于波士顿),HarvardUniversityisoneofthebestuniversitiesintheU.S.2.Please__________(将问题提出来)atthenextmeeting.3.Itisessentialthattheseapplicationforms________(尽早寄出去).4.Histastesare_______(和我的相同).

635.Itishightimethatwe_____________(保护濒临灭绝的动物).试题详解1.答案:Located/SituatedinBoston[解析]1)词组:locate/situatein(位于,坐落于)2)分词:此处应使用过去分词。2.答案:bringtheproblemforward[解析]词组:bringforward(提出)3.答案:besentasearlyaspossible[解析]主句中的表语是essential,谓语动词应使用“(should)+动词原形”的虚拟形式,同时使用被动语态。4.答案:inharmonywithmine[解析]词组:inharmonywith(和……相同)5.答案:protectedtheendangeredanimals[解析]1)虚拟语气:假设性虚拟语气句型itis(high)time(that)wedidsth.,that引导的从句中谓语动词应使用过去式。2)词汇:endangered(有危险的)

当前文档最多预览五页,下载文档查看全文

此文档下载收益归作者所有

当前文档最多预览五页,下载文档查看全文
温馨提示:
1. 部分包含数学公式或PPT动画的文件,查看预览时可能会显示错乱或异常,文件下载后无此问题,请放心下载。
2. 本文档由用户上传,版权归属用户,天天文库负责整理代发布。如果您对本文档版权有争议请及时联系客服。
3. 下载前请仔细阅读文档内容,确认文档内容符合您的需求后进行下载,若出现内容与标题不符可向本站投诉处理。
4. 下载文档时可能由于网络波动等原因无法下载或下载错误,付费完成后未能成功下载的用户请联系客服处理。
关闭